Chapter 21: Visual and Auditory Problems

Pataasin ang iyong marka sa homework at exams ngayon gamit ang Quizwiz!

The nurse, who is reinforcing medication teaching before administering the scheduled dose of pilocarpine, would include which statement? A. "You will need someone to drive you home." B. "This medication should be used as needed to reduce eye pain." C. "Eye irritation is to be expected during the first two weeks of use." D. "This medication will help to raise intraocular pressure to a near normal level."

A. "You will need someone to drive you home." Rationale Pilocarpine causes blurred vision and difficulty focusing, so it is important not to engage in any activities requiring visual acuity until the vision clears. Pilocarpine will not reduce eye pain, will not cause eye irritation, and will decrease, not increase, intraocular pressure. Test-Taking Tip: Start by reading each of the answer options carefully. Usually at least one of them will be clearly wrong. Eliminate this one from consideration. Now you have reduced the number of response choices by one and improved the odds. Continue to analyze the options. If you can eliminate one more choice in a four-option question, you have reduced the odds to 50/50. While you are eliminating the wrong choices, recall often occurs. One of the options may serve as a trigger that causes you to remember what a few seconds ago had seemed completely forgotten. p. 381

Which treatment is administered to fight infection in a patient with external otitis? A. Antibiotics B. Corticosteroids C. Mild analgesics D. Irrigation of the canal

A. Antibiotics Rationale Inflammation or infection of the epithelium of the auricle and ear canal are the main symptoms of external otitis. Topical or systemic antibiotics are prescribed to treat infection associated with external otitis, depending on whether surrounding tissue is involved. To control the inflammation, the health care provider generally prescribes corticosteroids. Mild analgesic drops usually control the pain. Irrigation of the canal with body temperature solutions softens the cerumen. p. 383

A patient has a prescription to receive timolol two drops to both eyes every 12 hours. The nurse would withhold the dose and question the prescription if the patient had which condition? A. Asthma B. Urinary retention C. Cluster headaches D. Chronic constipation

A. Asthma Rationale Timolol is a nonselective beta-adrenergic blocking agent that can cause bronchoconstriction and bronchospasm. For this reason, it should not be used in patients with severe chronic obstructive pulmonary disease (COPD) or asthma. Timolol will not increase urinary retention; that commonly is seen with use of anticholinergics. It will not cause or worsen headaches or constipation. p. 381

A patient is being provided with discharge instructions after undergoing cataract extraction and intraocular lens implantation. What should the nurse include in the teaching? A. Avoid straining during bowel movements. B. Keep consuming a full-liquid diet for 24 hours. C. Refrain from reading or watching TV for at least 12 hours. D. Keep a patch over the affected eye until the follow-up appointment with the surgeon.

A. Avoid straining during bowel movements. Rationale After cataract surgery, coughing, bending at the waist, and straining during bowel movements should be avoided because these activities increase intraocular pressure. It is not necessary to maintain dietary restrictions, refrain from reading or watching TV, or wear a patch. p. 376

A patient is having a surgical procedure that involves using extreme cold to create an inflammatory response to produce a sealing scar. What procedure will the nurse educate the patient regarding? A. Cryopexy B. Scleral buckling C. Pneumatic retinopexy D. Laser photocoagulation

A. Cryopexy Rationale Cryopexy is a procedure used to seal retinal breaks. This procedure involves using extreme cold to create the inflammatory reaction that produces the sealing scar. Scleral buckling is an extraocular surgical procedure that involves indenting the globe so that the pigment epithelium, the choroid, and the sclera move toward the detached retina. It involves suturing a silicone implant against the sclera. Pneumatic retinopexy is the intravitreal injection of a gas to form a temporary bubble in the vitreous that closes retinal breaks and provides apposition of the separated retinal layers. Laser photocoagulation involves using an intense, precisely focused light beam to create an inflammatory reaction. p. 377

A patient admitted to the hospital has been taking pilocarpine eye drops at home. What is the desired effect of this medication? Select all that apply. A. Facilitates aqueous humor outflow B. Reverses damage to the optic nerve C. Improves the patient's vision in dim light D. Lessens the amount of pupillary dilation E. Decreases the amount of fluid within the eye

A. Facilitates aqueous humor outflow D. Lessens the amount of pupillary dilation Rationale Pilocarpine is a cholinergic (parasympathomimetic) medication used to treat chronic open-angle glaucoma. It causes miosis (pupillary constriction), which improves the flow of the fluid (aqueous humor) within the trabecular meshwork of the eye. This keeps the pressure within the eye low and decreases likelihood of optic nerve damage. Damage to the optic nerve from glaucoma cannot be reversed with treatment. Miotic effects of the medication do not allow for pupil dilation that normally occurs in dim light. Pilocarpine does not decrease the amount of aqueous humor production as do some other medications (beta-adrenergic blockers) used to treat glaucoma. p. 381

The nurse is teaching a patient about managing blepharitis. The most important intervention for the patient with blepharitis is which of these? A. Gently cleaning the lid margins with baby shampoo. B. Monitoring the spread of infection to the opposing eye. C. Regular instillation of artificial tears to the affected eye. D. Teaching the patient and family members good hygiene techniques.

A. Gently cleaning the lid margins with baby shampoo. Rationale Blepharitis is a common chronic bilateral inflammation of the eyelid margins. Emphasize thorough cleaning practices of the skin and scalp. Gentle cleansing of the lid margins with baby shampoo can effectively soften and remove crusting. Blepharitis is not contagious nor does it spread unless conjunctivitis is occurring simultaneously. In this case, antibiotic drops may be used, but not artificial tears. It is not necessary to teach the family good hygiene, unless they are touching the eyelids. Test-Taking Tip: On a test day, eat a normal meal before going to school. If the test is late in the morning, take a high-powered snack with you to eat 20 minutes before the examination. The brain works best when it has the glucose necessary for cellular function. pp. 370-371

Which condition is caused by a refractive error the eye? A. Myopia B. Cataract C. Glaucoma D. Conjunctivitis

A. Myopia

Which condition is caused by a refractive error in the eye? A. Myopia B. Cataract C. Glaucoma D. Conjunctivitis

A. Myopia Myopia is a refractive error of the eye characterized by the ability to see close objects clearly, whereas distant objects appear blurred. A cataract manifests as opacity of the lens, leading to decreased vision. It is not a refractive error. Glaucoma is a group of disorders characterized by increased intraocular pressure; it may lead to permanent blindness. Conjunctivitis is an infection of the conjunctiva caused by bacteria or viruses. Test-Taking Tip: Identify option components as correct or incorrect. This may help you identify a wrong answer. p. 367

Which auditory disorders are related to the inner ear? A. Presbycusis B. Cholesteatoma C. Serous otitis D. Otosclerosis

A. Presbycusis Rationale Presbycusis is hearing loss associated with aging; it is related to the inner ear. Cholesteatoma, serous otitis, and otosclerosis are disorders of the middle ear. p. 391

In reinforcing health teaching to a patient diagnosed with primary open-angle glaucoma, the nurse would include which information about the disorder? A. Pressure damage to the optic nerve may occur because of clogged drainage channels. B. The retinal nerve is damaged by an abnormal increase in the production of aqueous humor. C. The pupillary opening is blocked secondary to decreased aqueous humor in the anterior chamber. D. The lens enlarges with normal aging, pushing the iris forward, blocking the outflow of aqueous humor.

A. Pressure damage to the optic nerve may occur because of clogged drainage channels. Rationale With primary open-angle glaucoma, there is increased intraocular pressure because the aqueous humor cannot drain properly from the eye. This leads to damage to the optic nerve over time. The iris is not affected, the retinal nerve is not damaged, and the aqueous humor builds up because of blockage. p. 379

A student nurse is assisting a patient who is blind using a sighted-guide technique. Which action by the student nurse requires immediate intervention? A. Walking behind the patient holding the patient's back B. Describing the environment to the patient while walking C. Helping the patient to sit by placing the patient's hand on the seat of the chair D. Standing slightly in front and to one side of the patient and providing elbow for support

A. Walking behind the patient holding the patient's back Rationale While assisting a blind patient using sighted-guide technique, the nurse should walk slightly ahead of the patient, with the patient holding the back of the nurse's arm. This action will help the blind patient to walk easily. The nurse should describe the environment while walking to help orient the patient. The student nurse should help the patient sit by placing one of his or her hands on the seat of the chair. The nurse should stand slightly in front and to one side of the patient and provide an elbow for the patient to hold. pp. 368-370

The patient calls the clinic about a sty that the patient has had for some time on the upper eyelid. The patient says warm moist compresses have been used, but it is no better. What should the nurse tell the patient to do? A. "Go to the pharmacy to get some eye drops." B. "Come in so the ophthalmologist can remove the lesion for you." C. "The health care provider will need to inject it with an antibiotic." D. "Wash the lid margins with baby shampoo to remove the crusting."

B. "Come in so the ophthalmologist can remove the lesion for you." Rationale A chalazion may evolve from a sty, or hordeolum, as it did for this patient. Initial treatment is with warm compresses, but when they are ineffective, the lesion may be surgically removed or injected with corticosteroids. Washing the lid margins with baby shampoo is done with blepharitis. p. 370

While examining the eye of a patient, the nurse finds a cone-shaped anterior cornea. Which other finding is associated with this condition? A. Pain B. Blurred vision C. Corneal inflammation D. Abnormal color perception

B. Blurred vision Rationale Keratoconus is a noninflammatory eye disorder in which the anterior cornea thins and protrudes forward, taking on a cone shape. The only symptom associated with keratoconus is blurred vision. Pain is a symptom of corneal ulcer. Corneal inflammation is a symptom of keratitis. Abnormal color perception is associated with cataracts. Test-Taking Tip: Multiple-choice questions can be challenging because students think that they will recognize the right answer when they see it or that the right answer will somehow stand out from the other choices. This is a dangerous misconception. The more carefully the question is constructed, the more each of the choices will seem like the correct response. p. 373

The nursing instructor is teaching a group of nursing students about the treatment of cerumen and foreign bodies in the external ear canal. Which statements made by the nursing student indicate an understanding regarding the instruction? Select all that apply. A. "The canal should be irrigated with cold solutions." B. "The ear canal should be completely occluded with the syringe tip." C. "Use of cotton-tipped applicators to clean the ears should be avoided." D. "Mild lubricant drops should be used as initial treatment for cerumen removal." E. "Mineral oil can be used with microscopic guidance to kill an insect before removal."

C. "Use of cotton-tipped applicators to clean the ears should be avoided." E. "Mineral oil can be used with microscopic guidance to kill an insect before removal." Rationale Mineral oil or lidocaine drops can be used to kill an insect before removal with microscope guidance. Ears should be cleaned with a washcloth and finger, and never with cotton-tipped applicators. This is because penetration of the middle ear by a cotton-tipped applicator can cause serious injury to the tympanic membrane (TM) and ossicles, and can also cause cerumen to become impacted against the TM and impair hearing. Management of cerumen involves irrigation of the canal with body temperature solutions to soften the cerumen. When irrigating the ear canal to soften the cerumen, it is important to make sure that the ear canal is not completely occluded with the syringe tip. Mild lubricant drops may be used to soften the earwax only if irrigation does not remove the wax, and not as the initial step for cerumen removal. p. 384

A patient has received a prescription for ketoconazole eyedrops. The nurse recognizes that the probable reason for the medication is that the patient is experiencing what? A. Myopia B. Astigmatism C. Acanthamoeba keratitis D. Generalized poor hygiene of the eye

C. Acanthamoeba keratitis Rationale Ketoconazole is an antifungal eyedrop that is prescribed for the treatment of Acanthamoeba keratitis because the causative organism is resistant to other drugs. Myopia is an inability to accommodate for objects at a distance; it is not a fungal infection. Astigmatism is caused by an irregular corneal curvature and is not a fungal infection. Ketoconazole is not used for general hygiene of the eye. p. 372

The nurse cares for a patient with a detached retina. The patient says, "Before my eye was patched, I saw a lot of spots." The nurse explains that the symptoms are caused by what? A. Contamination of the aqueous humor B. Pieces of the retina floating within the eye C. Blood cells released into the eye by the detached retina D. Spasms of the retinal blood vessels traumatized by the detached retina

C. Blood cells released into the eye by the detached retina Rationale The spots commonly reported by patients with retinal detachment are blood cells released into the vitreous humor in the detachment. These are also referred to as "floaters." Contamination, retinal fragments, and spasms of the retinal blood vessels are not the cause of floaters in the field of vision of a person with a detached retina. pp. 376-377

The nurse is performing an eye assessment and determines the pupils are 1 to 2 mm. What medication taken by the patient does the nurse recognize may be causing this symptom? A. Carteolol B. Dipivefrin C. Carbachol D. Latanoprost

C. Carbachol Rationale Carbachol is a cholinergic agent that stimulates iris sphincter contraction and results in miosis. Carteolol is a nonselective beta-adrenergic blocker and decreases intraocular pressure but does not cause miosis. Dipivefrin is a sympathomimetic agent that decreases aqueous humor production but does not cause miosis. Latanoprost is a prostaglandin F2-alpha analog that does not stimulate contraction of the iris sphincter; therefore, it does not cause miosis. p. 381

A patient reports ocular pain, photophobia, decreased visual acuity, headaches, corneal edema, and conjunctiva that is reddened and swollen. The nurse recognizes that the patient is most likely experiencing which type of inflammation? A. Scotoma B. Blepharitis C. Endophthalmitis D. Cytomegalovirus retinitis

C. Endophthalmitis Rationale Endophthalmitis is an extensive intraocular inflammation of the vitreous cavity. Ocular pain, photophobia, decreased visual acuity, headaches, reddened, swollen conjunctiva, and corneal edema are the symptoms of endophthalmitis. Blepharitis is a common chronic bilateral inflammation of the lid margins. Cytomegalovirus retinitis is an opportunistic infection that occurs in patients who are immunocompromised. Scotoma is presence of blind spots in the visual field; it is not an infection. p. 382

Which extraocular eye disorder will the nurse suspect in the patient demonstrating inflammation of the cornea and exophthalmos? A. Strabismus B. Keratoconus C. Exposure keratitis D. Keratoconjunctivitis sicca

C. Exposure keratitis Rationale Exposure keratitis is an inflammation or infection of the cornea that occurs when the patient has exophthalmos due to thyroid disease or masses behind the globe. Therefore the nurse will suspect exposure keratitis. Strabismus is not associated with inadequately closed eyelids, and there is no inflammation. Instead the patient with strabismus cannot consistently focus the two eyes on same object simultaneously. Keratoconus is a noninflammatory condition in which the patient experiences blurred vision. Keratoconjunctivitis sicca is a condition of dry eyes, particularly seen in older adults and individuals with certain systemic diseases such as scleroderma and systemic lupus erythematosus. p. 372

Which nursing intervention is most appropriate for facilitating communication with a patient who has a hearing impairment? A. Speaking loudly and shouting if necessary B. Asking the patient questions that can be answered with a yes or no response C. Standing close to the patient and speaking slowly and clearly in a normal tone D. Standing to one side of the patient when speaking and directing the voice directly into the patient's ear

C. Standing close to the patient and speaking slowly and clearly in a normal tone Rationale Standing close to and directly in front of the patient will greatly facilitate communication. The nurse also should ensure that the patient can see the nurse's mouth to help facilitate lip-reading. Shouting at the patient with a hearing impairment distorts the voice and further hinders understanding. Asking yes-or-no questions and standing to one side and speaking directly into the patient's ear are not appropriate or effective means of communicating with the patient who has a hearing impairment. p. 390

Which hearing-impaired patient is the most ideal candidate for a cochlear implant? A. The middle-aged patient being treated for benign paroxysmal positional vertigo (BPPV) B. The older adult with slight hearing loss which does not significantly impact activities C. The teen with profound sensorineural hearing loss after tympanic membrane (TM) rupture D. The construction worker with tinnitus after prolonged exposure to excessively loud equipment

C. The teen with profound sensorineural hearing loss after tympanic membrane (TM) rupture Rationale The teenager with profound sensorineural hearing loss after TM rupture is the ideal candidate for a cochlear implant, as hearing loss occurred after speech development, and this patient will adapt to the implant well. Patients who suffer from BPPV typically do not demonstrate hearing loss. A cochlear implant is not indicated for slight hearing loss at any age. Tinnitus is the perception of sound when no source is present. A cochlear implant would be inappropriate treatment for tinnitus. pp. 390-391

A patient asks the nurse why the lights are being dimmed prior to the instillation of pupil-dilating eye drops. What is the best response by the nurse? A. To decrease pain B. To prevent anxiety C. To minimize photophobia D. To minimize intraocular pressure

C. To minimize photophobia Rationale Pupil dilation medications enlarge the pupil during eye examinations. After administering pupil dilation medications, patients generally have photophobia. Analgesics are administered to decrease pain. Anxiolytics are given to patients to prevent anxiety. Miotics and oral or intravenous hyperosmotic agents such as glycerin liquid, isosorbide solution, and mannitol solution are useful in lowering the intraocular pressure. p. 374

The nurse provides discharge instructions to a patient with glaucoma. Which statement by the patient indicates understanding of the teaching? A. "I'll limit my fluid intake." B. "I'll change positions slowly." C. "I'll use my eye drops until my vision clears." D. "I'll check the labels on my nonprescription drugs."

D. "I'll check the labels on my nonprescription drugs." Rationale Nonprescription drugs, even caffeine, may increase intraocular pressure, resulting in enough pressure to cause damage. Therefore it is important for the patient to check the contents of all drug labels. Limitation of fluid intake and slow position changes will not affect intraocular pressure. Eye drop medications for glaucoma must be taken as prescribed. Stopping these medications could cause a rebound increase in intraocular pressure. p. 379

When performing teaching with a patient with glaucoma while administering a scheduled dose of pilocarpine, the nurse would include which statement? A. "Prolonged eye irritation is an expected adverse effect of this medication." B. "This medication will help to raise intraocular pressure to a near normal level." C. "This medication needs to be continued for at least five years after the initial diagnosis." D. "It is important not to do activities requiring visual acuity immediately after administration."

D. "It is important not to do activities requiring visual acuity immediately after administration." Rationale Pilocarpine causes blurred vision and difficulty in focusing, so it is important not to engage in any activities requiring visual acuity until the vision clears. Prolonged eye irritation is not an expected adverse effect of pilocarpine. Pilocarpine will stimulate iris sphincter contraction. At least five years is not an appropriate amount of time to estimate to the patient. p. 381

After providing education to a patient with otosclerosis, the nurse determines that the teaching was effective when the patient states that the hearing loss is due to what? A. A tumor in the middle ear B. Too much ear wax buildup C. Taking too much aspirin over the years D. A problem with the bones in the middle ear

D. A problem with the bones in the middle ear Rationale Otosclerosis, the deterioration of the bones of the middle ear, is the most common cause of conductive hearing loss. Tumors, too much ear wax, and too much aspirin are problems that may affect hearing but are not related to otosclerosis. p. 385

A patient with poor visual acuity is diagnosed with age-related macular degeneration (AMD). Which nursing intervention should be the nurse's priority? A. Teach about visual enhancement techniques. B. Teach nutritional strategies to improve vision. C. Assess coping strategies and support systems. D. Assess impact of vision on normal functioning.

D. Assess impact of vision on normal functioning. Rationale The most important nursing intervention is to assess the patient's ability to function with the visual impairment. The nurse will use this information to plan nursing care, including assessment of the patient's coping strategies and teaching about vision enhancement techniques and nutrition. pp. 378-379

A patient sustained an eye injury, and the nurse assesses blood in the anterior chamber as well as redness of the sclera. What is a priority nursing action for this patient? A. Applying pressure on the eye B. Giving oral fluids to the patient C. Instructing the patient to blow the nose D. Elevating the head of bed to 45 degrees

D. Elevating the head of bed to 45 degrees Rationale Eye injuries may be caused due to trauma, foreign bodies, chemical burns, or thermal burns and can be a serious threat to vision if not treated appropriately. The assessment findings include pain, photophobia, redness, swelling, and blood in the anterior chamber of the eye. Elevating the head of the patient's bed to 45 degrees helps minimize edema and swelling, thereby preventing the obstruction of vision. The nurse should avoid applying pressure on the eye. Oral fluids and food should not be given to the patient. The patient should be instructed not to blow the nose because it may cause black eyes. p. 371

What is a possible effect of cold otic drops administered in the ear canal? A. Vertigo B. Infection C. Barotrauma D. Tympanum burn

A. Vertigo Rationale Administration of cold ear drops can cause vertigo due to the stimulation of the semicircular canals of the ear. Hot drops, not cold drops, can burn the tympanic membrane of the ear. For this reason, ear drops are instilled at room temperature. Barotrauma of the ear is ear discomfort caused by pressure changes. Increased air or water pressure can cause this. Microorganisms such as bacteria or fungi cause infection to the ear. Eardrops are not responsible for this. Test-Taking Tip: Cold eardrops can stimulate the semicircular canals of the ear. This does not change the pressure in the ear or make any change in the microorganism population present in the ear. p. 383

The nurse is caring for a 25-year-old sexually active woman who was just vaccinated against the rubella virus. Which instruction should the nurse give this patient? A. "Refrain from becoming pregnant for at least three months after being immunized." B. "Refrain from exposure to chemicals such as toluene, carbon disulfide, and mercury." C. "Undertake serological testing for the presence of antibodies against the rubella virus." D. "Be aware that rubella infection during the first eight weeks of pregnancy is associated with an 85 percent incidence of congenital rubella syndrome."

A. "Refrain from becoming pregnant for at least three months after being immunized." Rationale The nurse should instruct the patient to avoid pregnancy for at least three months after being immunized against the rubella virus, which can cause deafness in the fetus and malformations affecting the ear. A patient should be vaccinated against rubella virus only when serological tests for the presence of antibodies for this virus are found positive. Chemicals such as carbon disulfide are ototoxic and may damage the inner ear. Such patients may exhibit signs and symptoms of ototoxicity. However, ototoxicity is not related to rubella infection. Rubella infection during the first eight weeks of pregnancy is associated with an 85 percent incidence of congenital rubella syndrome. This syndrome may cause sensorineural deafness in the child. This information is, however, not significant for a woman who is already vaccinated against rubella virus. p. 389

The patient has been diagnosed with benign paroxysmal positional vertigo. The nurse knows that which anatomic area of the ear contributes to this disturbance? A. 1: semicircular canals B. 2: oval window C. 3: facial nerve D. 4: cochlea

A. 1: semicircular canals Benign paroxysmal positional vertigo occurs when the organs of balance (the three semicircular canals (1)) have debris or excessive pressure within the lymphatic fluid. The oval window (2) is in the middle ear and receives the vibrations of the ossicles. The facial nerve (CN VII) (3) traverses above the oval window in the middle ear and may be damaged by chronic ear infection or trauma that causes problems related to voluntary facial movements. The cochlea (4) is the coiled structure in the inner ear that is the receptor organ for hearing. p. 387

An acoustic neuroma is removed from a patient, and the nurse instructs the patient about tumor recurrence. What should the nurse instruct the patient to monitor? Select all that apply. A. Episodes of dizziness B. Lack of coordination C. Worsening of hearing D. Inability to close the eye E. Clear drainage from the nose

A. Episodes of dizziness C. Worsening of hearing D. Inability to close the eye E. Clear drainage from the nose Rationale An acoustic neuroma is a unilateral benign tumor that occurs where the vestibulocochlear nerve (CN VIII) enters the internal auditory canal. Clinical manifestations of tumor recurrence, including facial nerve (CN VII) paralysis, can be manifested by intermittent vertigo, hearing loss, and inability to close the eye. The patient should report any clear, colorless discharge from the nose because it may be cerebrospinal fluid (CSF), which increases the risk of infection. Lack of coordination is not a manifestation of acoustic neuroma. p. 387

The nursing student prepares a chart on the diagnostic tests and treatments associated with different auditory problems. Which disease has the nursing student correctly associated with its diagnostic test and treatment? A. External otitis B. Chronic otitis media C. Endolymphatic hydrops D. Otitis media with effusion

A. External otitis Rationale External otitis involves inflammation or infection of the epithelium of the auricle and ear canal. Pseudomonas aeruginosa is the most common bacterial cause of this disease. Culture and sensitivity studies of the drainage can be used to diagnose external otitis. Topical treatments may include antibiotic drops for infection and corticosteroids for inflammation. Chronic otitis media can involve the mastoid bone, so mastoid x-ray is one of the diagnostic tests for this disease. However, a labyrinthectomy is not used for treating chronic otitis media; instead, it can be used as surgical therapy for Ménière's disease. In otosclerosis, tuning fork tests and an audiogram demonstrate a difference of at least 20 to 25 dB between air and bone conduction levels of hearing. Otosclerosis can be treated surgically by replacing the stapes with a metal or Teflon prosthesis. Endolymphatic hydrops, or Ménière's disease, is not diagnosed by the air-bone gap, nor treated with a prosthetic stapes. For the diagnosis of Ménière's disease, tests are done to rule out CNS disease; it is treated with antihistamines and benzodiazepines during an acute attack. This is not how otitis media with effusion is diagnosed or treated. pp. 383-384

The nurse provides education to a patient with Ménière's disease. What long-term effect of the disease should the nurse include in the teaching? A. Hearing loss B. Double vision C. Chronic vertigo D. Chronic headaches

A. Hearing loss Rationale Ménière's disease, which involves the inner ear, is characterized by episodes of acute vertigo and tinnitus. It can result in progressive and irreversible sensorineural hearing loss. Double vision is not an associated symptom of Ménière's disease. Chronic vertigo and chronic headaches may or may not be associated with Ménière's disease. p. 386

What instruction should the nurse include when teaching a caregiver ways to communicate better with a patient who has hearing loss? Select all that apply. A. Maintain eye contact. B. Speak normally and slowly. C. Move closer to the better ear. D. Overenunciate whatever you say. E. Write out names or difficult words. F. Do not draw attention with hand movements.

A. Maintain eye contact. B. Speak normally and slowly. C. Move closer to the better ear. E. Write out names or difficult words. Rationale The family and caregivers can help the patient with hearing loss by using strategies for improving verbal and nonverbal communication. These strategies include maintaining eye contact when talking, speaking normally and slowly, moving closer to the better ear, and writing out names and words that are difficult to understand. Hand movements can be used to draw attention. Overenunciating should be avoided. Test-Taking Tip: Look for answers that focus on the client or are directed toward feelings. p. 390

A nurse is explaining measures to prevent external otitis to a group of patients. Which interventions should the nurse include in the teaching? Select all that apply. A. Report any itching in the ear. B. Use earplugs when swimming. C. Keep the ears as dry as possible. D. Avoid putting anything in the ear canal. E. Avoid using hairdryers to dry the external ear.

A. Report any itching in the ear. B. Use earplugs when swimming. C. Keep the ears as dry as possible. D. Avoid putting anything in the ear canal. Rationale External otitis is associated with the inflammation or infection of the epithelium of the auricle and ear canal. External otitis can be prevented by keeping the ears as dry as possible because moisture promotes bacterial growth. However, if itching becomes a problem, it should be immediately reported. The patient should be advised to avoid inserting anything into the ear, because this may cause infection. Water in the swimming pool may be contaminated or contain chemicals that alter the flora of the external canal. Therefore earplugs should be used at the time of swimming. A hair dryer set to low and held at least six inches from the ear can speed water evaporation. p. 384

A patient receiving injectable gentamicin for acute diarrhea complains of tinnitus, diminished hearing, and dizziness. What is the most appropriate nursing action? A. Stop the drug immediately. B. Decrease the drug dosage. C. Plug the ear canals with cotton. D. Reassure the patient that the effects are temporary.

A. Stop the drug immediately. Rationale Gentamicin is an aminoglycoside drug and is an ototoxic drug, and the nurse should monitor the patient for ototoxic effects, such as tinnitus, diminished hearing, and changes in equilibrium. If the patient experiences these symptoms, the drug should be immediately stopped. The symptoms may reverse after discontinuing the drug. Decreasing the dosage may still be ototoxic. Reassuring the patient that symptoms may be temporary and plugging the ears may not contribute to management of symptoms. p. 387

A 25-year-old patient reports tinnitus, diminished hearing, and changes in equilibrium. The nurse assesses that the patient, a worker in a rayon fiber-manufacturing unit, has been exposed to carbon disulfide. What would the nurse conclude from this assessment? A. The patient has ototoxicity. B. The patient has Ménière's disease. C. The patient has central hearing loss. D. The patient has an acoustic neuroma.

A. The patient has ototoxicity. Rationale Chemicals used in industry such as carbon disulfide may damage the inner ear. The patient exposed to ototoxic chemicals may exhibit signs and symptoms of ototoxicity such as tinnitus, diminished hearing, and changes in equilibrium. Ménière's disease is characterized by symptoms caused by inner ear disease that include episodic vertigo, tinnitus, fluctuating sensorineural hearing loss, and aural fullness. However, symptoms usually begin between 30 and 60 years of age. Acoustic neuroma is seen in patients between 40 and 60 years of age. This patient is too young for acoustic neuroma. It is a unilateral benign tumor that occurs where the cranial nerve VIII enters the internal auditory canal. Central hearing loss involves an inability to interpret sound, including speech. It occurs due to a problem in the central nervous system (CNS) but is not manifested as any changes in equilibrium. p. 389

The patient experiences loss of high-pitched sounds but understands speech with good response to sound amplification. Which statement accurately describes the patient's condition? A. There is loss of sensory hair cells. B. There is stiffening of the basilar membrane. C. There are degenerative changes in the cochlea. D. There is atrophy of the blood vessels in the wall of the cochlea.

A. There is loss of sensory hair cells. Rationale Presbycusis is hearing loss associated with aging. Loss of sensory hair cells occurs in the case of neural presbycusis; as a result, the patient experiences loss of high-pitched sounds. The patient, however, correctly understands speech and responds well to sound amplification. Stiffening of the basilar membrane occurs with cochlear presbycusis, and this condition interferes with sound transmission in the cochlea. With this disorder, speech discrimination is affected with higher-frequency losses. Degenerative changes in the cochlea, termed as neural presbycusis, results in loss of speech discrimination. In the case of metabolic presbycusis, there is atrophy of the blood vessels in wall of the cochlea. Such patients experience uniform loss for all frequencies accompanied by recruitment. p. 391

Which statement is true regarding a patient with conductive hearing loss? A. This patient hears better in a noisy environment. B. This patient can hear sound, but not understand speech. C. This patient speaks very loudly because of an inability to hear his or her own voice. D. This patient does not benefit from using a hearing aid because it makes speech clearer, not louder.

A. This patient hears better in a noisy environment. Rationale A patient with a conductive hearing loss hears better in a noisy environment. A patient with sensorineural hearing loss hears sound but does not understand speech. A patient with conductive hearing loss often speaks softly because his or her own voice seems loud. If correcting the cause of conductive hearing loss is not possible, a hearing aid may help if the loss is greater than 40 to 50 dB. For a patient with sensorineural hearing loss, the hearing aid may not always help because it only makes sounds and speech louder, not clearer. p. 387

The nurse is interviewing an elderly patient who has hearing loss. Which strategies would best assist the nurse in communicating with this patient? Select all that apply. A. Use simple sentences. B. Overenunciate speech. C. Speak normally and slowly. D. Exaggerate facial expressions. E. Avoid distracting environments. F. Raise the voice to a higher pitch.

A. Use simple sentences. C. Speak normally and slowly. E. Avoid distracting environments. Rationale Speak normally and slowly directly into the patient's better ear. Use simple sentences, rephrase sentences, or use different words. Write out names or difficult words. Avoid shouting and distracting environments. Overenunciating the speech, exaggerating facial expressions, and raising the voice to a higher pitch are not effective strategies for communicating with the patient with hearing loss. p. 390

The nurse assesses a patient with an inner ear problem. What is the most significant symptom for which the nurse should be alert in this patient? A. Vertigo B. Tinnitus C. Headaches D. Hearing loss

A. Vertigo Rationale The patient with an inner ear problem will most often complain of vertigo, because the inner ear is directly related to maintenance of equilibrium and balance. The patient should be assessed by the nurse because of the safety implications. A patient with an inner ear issue may manifest tinnitus (ringing or buzzing in the ear), headaches, or hearing loss. Vertigo presents the greatest safety risk. p. 386

A patient is diagnosed with proliferative diabetic retinopathy and is scheduled for a surgical procedure. Which surgical procedure will be used to relieve traction on the retina and will the nurse educate the patient? A. Vitrectomy B. Cryotherapy C. Photodynamic therapy D. Ocular coherence tomography

A. Vitrectomy

The nurse is providing care for a 73-year-old male patient who has sought care because of a loss in hearing acuity over the past several years. Which statement by the nurse is most accurate? A. "This is often caused by an infection that will resolve on its own." B. "Many people experience an age-related decline in their hearing." C. "This is likely an effect of your medications. Try stopping them for a few days." D. "You can likely accommodate for your hearing loss with a few small changes in your routine."

B. "Many people experience an age-related decline in their hearing." Rationale Presbycusis is a loss of hearing that is both common and age related. Infections most often are accompanied by different symptoms. It would be inappropriate to counsel the patient to stop medications. It would be simplistic to advise the patient to accommodate the hearing loss rather than seek intervention. p. 391

When using the otoscope, the nurse is unable to see the landmarks or light reflex of the tympanic membrane. The tympanic membrane is bulging and red. What does the nurse think is most likely occurring in the patient's ear? A. Swimmer's ear B. Acute otitis media C. Impacted cerumen D. Chronic otitis media

B. Acute otitis media Rationale The manifestations of the inability to see the landmarks or light reflex of the tympanic membrane and the bulging and redness of the tympanic membrane are those of acute otitis media. With swimmer's ear and chronic otitis media, there is frequently drainage in the external auditory canal. Impacted cerumen would block the visualization of the tympanic membrane. p. 384

The patient is given an oral dose of glycerol followed by serial audiograms over three hours. Which symptom did the nurse assess in the patient? A. Presence of positive Schwartz's sign B. Aural fullness and fluctuating sensorineural hearing loss C. Reduction in touch sensation in the posterior ear canal D. A painless condition with hearing loss, nausea, and episodes of dizziness

B. Aural fullness and fluctuating sensorineural hearing loss Rationale A glycerol test is a diagnostic test aimed to confirm Ménière's disease. The symptoms of this disorder include aural fullness and fluctuating sensorineural hearing loss. A series of audiograms over three hours is done after the patient is given an oral dose of glycerol. Improvement in hearing or speech discrimination occurs due to the osmotic effect of glycerol that pulls fluid from the inner ear. An otoscopic examination, the Rinne test, the Weber test, audiometry, and tympanometry are the diagnostic tests used to confirm otosclerosis. Such patients show positive Schwartz's sign. Patients with acoustic neuroma have reduced touch sensation in the posterior ear canal. Diagnostic tests to confirm acoustic neuroma include neurologic, audiometric, and vestibular tests; CT scans; and MRI. A mastoid x-ray is one of the tests used to diagnose chronic otitis media. This is a painless auditory disorder with hearing loss, nausea, and episodes of dizziness. p. 386

The patient with Ménière's disease had decompression of the endolymphatic sac to reduce the frequent and incapacitating attacks the patient was experiencing. What should the nurse include in the discharge teaching for this patient? A. Airplane travel will be more comfortable now B. Avoid sudden head movements or position changes C. Cough or blow the nose to keep the eustachian tube clear D. Take antihistamines, antiemetics, and sedatives for recovery

B. Avoid sudden head movements or position changes Rationale After ear surgery the patient should avoid sudden head movements or position changes. Airplane travel should be avoided at first, because increased pressure and ear popping normally are experienced, which will disrupt healing. The patient should not cough or blow the nose, because this increases pressure in the eustachian tube and middle ear cavity and may disrupt healing. Antihistamines, antiemetics, and sedatives are used to decrease the symptoms of acute attacks of Ménière's disease. p. 386

Which type of hearing loss involves an inability to interpret sound, including speech, because of a problem in the central nervous system (CNS)? A. Mixed hearing loss B. Central hearing loss C. Conductive hearing loss D. Sensorineural hearing loss

B. Central hearing loss Rationale Central hearing loss involves an inability to interpret sound, including speech, because of a problem in the CNS. Mixed hearing loss occurs due to a combination of conductive and sensorineural causes. In conductive hearing loss, the patient often speaks softly because hearing his or her own voice seems loud. Sensorineural hearing involves a diminished ability to hear high-pitched sounds, including consonants, because of the impairment of function of the inner ear or the vestibulocochlear nerve (CN VIII). pp. 387-388

A patient experiences an attack of Ménière's disease. Which intervention is most important for the nurse to include in the patient's plan of care? A. Increased fluid intake B. Decreased environmental stimuli C. Provision of the patient's regular diet D. Assessment for orthostatic hypotension

B. Decreased environmental stimuli Rationale The etiology of Ménière's disease is not well understood, but stress and excessive sensory stimulation are possible causes. Decreasing environmental stimuli is one approach to treatment and controlling the severity of the symptoms. Fluid intake should be decreased; this may ease the symptoms because in Ménière's disease there is an increase in the endolymphatic fluid of the inner ear. It is believed this causes disease symptoms. If there is no nausea and vomiting, the patient may eat a regular diet as tolerated, but this is not as high a priority as decreasing environmental stimuli is. Patients may experience tinnitus and vertigo, but the blood pressure is not affected, so orthostatic hypotension does not occur. p. 386

The nurse is reviewing the types of hearing loss. The patient who has a sensorineural hearing loss should display which of these problems? A. Hears better in a noisy environment B. Hears sound but does not understand speech C. Often speaks softly because his or her own voice seems loud D. Experiences clearer sound with a hearing aid if the loss is over 40 to 50 dB

B. Hears sound but does not understand speech Rationale The patient with sensorineural hearing loss has the ability to hear sound but not to understand speech. The ability to hear high-pitched sounds, including consonants, diminishes. Sounds become muffled and difficult to understand. A hearing aid may help some patients, but it only makes sounds and speech louder, not clearer. Hearing better in a noisy atmosphere, speaking softly because one's own voice seems loud, and hearing clearer with a hearing aid if the loss is 40 to 50 dB are characteristics of conductive hearing loss. p. 387

Which criteria should the nurse consider for the placement of an implanted hearing system in a patient to treat moderate to severe sensorineural hearing loss? Select all that apply. A. Loss of sensory hair cells B. Normal middle ear anatomy C. Bilateral hearing impairment D. Normally functioning Eustachian tube E. Stable bilateral sensorineural hearing loss

B. Normal middle ear anatomy D. Normally functioning Eustachian tube E. Stable bilateral sensorineural hearing loss Rationale An implanted hearing system treats moderate to severe sensorineural hearing loss. Criteria for placement of this device include normal middle ear anatomy, a normally functioning Eustachian tube, and stable bilateral sensorineural hearing loss. Loss of sensory hair cells leads to sensory presbycusis that responds well to sound amplification. Patients with bilateral hearing impairment require binaural hearing. Test-Taking Tip: Be alert for details about what you are being asked to do. In this question type, you are asked to select all options that apply to a given situation or patient. All options likely relate to the situation, but only some of the options may relate directly to the situation. p. 390

A nursing student is preparing a chart on the various problems associated with different parts of the ear. Which chart entry indicates that the nursing student has correctly mapped the disease with the part of the ear involved and the cause of the disease? A. BPPV B. Otosclerosis C. Ménière's disease D. Chronic otitis media

B. Otosclerosis Rationale Otosclerosis affects the internal ear. It is a hereditary autosomal dominant disease and therefore caused by genetic factors. BPPV is an infection of the inner ear, not the middle ear. Head trauma, infection, or degeneration from the aging process injures the utricle, which leads to the formation of debris known as "ear rocks." This debris leads to BPPV by causing vertigo with specific head movements. Ménière's disease does affect the inner ear. However, its cause is unknown; excessive accumulation of endolymph in the membranous labyrinth is an effect of this disease, not its cause. Chronic otitis media affects the middle ear, not the inner ear. Also, it is caused by repeated attacks of otitis media, especially in adults who have a history of recurrent otitis in childhood. pp. 385-386

Which type of presbycusis is associated with the loss of sensory hair cells? A. Neural B. Sensory C. Cochlear D. Metabolic

B. Sensory Rationale Sensory presbycusis is associated with atrophy of the auditory nerve and loss of sensory hair cells. Neural presbycusis is associated with degenerative changes in the cochlea and spinal ganglion. Cochlear presbycusis is associated with stiffening of basilar membrane. Metabolic presbycusis is associated with atrophy of the blood vessels in the wall of cochlea. p. 391

What instructions should the nurse include when teaching a patient how to administer ear drops? Select all that apply. A. Administer cold, not warm, drops. B. The tip of the dropper should not touch the ear. C. The ear should be positioned so that the drops can run into the canal. D. The drops should not be put in using a cotton wick placed in the ear canal. E. The position of the ear should be maintained for two minutes to let the drops spread.

B. The tip of the dropper should not touch the ear. C. The ear should be positioned so that the drops can run into the canal. E. The position of the ear should be maintained for two minutes to let the drops spread. Rationale When administering ear drops, the patient should position the ear so that the drops run into the canal, and this position should be maintained for two minutes to let the drops spread. The dropper should not touch the ear; avoiding contact reduces the spread of infection. The ear drops should be at room temperature when administered. Cold drops can cause vertigo; very warm drops can burn the tympanic membrane. Sometimes the drops are placed onto a wick of cotton that is placed in a canal. pp. 383-384

Which instruction given by the nursing student during ear care training needs correction by the registered nurse? A. "Do not put anything in your ear canal." B. "Report itching if it becomes a problem." C. "Dry the ear with cotton-tipped applicators." D. "Report chronic excessive cerumen if it impairs your hearing."

C. "Dry the ear with cotton-tipped applicators." Rationale The ears should be kept as dry as possible. However, cotton-tipped applicators should not be used for drying the ears. Rather, a hair dryer set to low and held at least 6 inches from the ear can be used to evaporate the water from the ear. Unless requested by the health care provider, nothing should be put in the ear canal. The health care provider should be consulted if consistent itching occurs or if excessive cerumen impairs hearing. p. 384

A patient has just been fitted with a new hearing aid. Which statement by the patient reflects a correct understanding of care and use of hearing aids? Select all that apply. A. "I will replace the batteries monthly." B. "I need to clean the ear molds daily." C. "I will disconnect or remove the batteries when not in use." D. "I will always keep them in their storage box when I am not wearing them." E. "I will wear this at home at first to get used to how it works while it is quiet."

C. "I will disconnect or remove the batteries when not in use." D. "I will always keep them in their storage box when I am not wearing them." E. "I will wear this at home at first to get used to how it works while it is quiet." Rationale Initially, use of the hearing aid should be restricted to quiet situations in the home. As adjustment to the increase in sounds and background noise occurs, the patient can progress to situations in which several people will be talking simultaneously. Next, the environment can be expanded to the outdoors and then the shopping mall or grocery store. When the hearing aid is not being worn, it should be disconnected or removed and kept in its storage box. Battery life averages one week. Ear molds should be cleaned weekly or as needed. pp. 389-390

During assessment, the nurse finds rough sandpaper-like changes to the upper border of the auricle of the patient. What does this observation indicate? A. Otosclerosis B. Cholesteatoma C. Actinic keratoses D. Chronic otitis media

C. Actinic keratoses Rationale Actinic keratoses are premalignant lesions associated with chronic sun exposure. The upper border of the auricle with premalignant lesions looks like rough sandpaper. Otosclerosis is a hereditary autosomal dominant disease that results in hearing loss and mostly affects young adults. Cholesteatoma is the mass of epithelial cells and cholesterol that develops in the middle ear, which may enlarge and destroy the adjacent bones. Chronic otitis media is a painless condition of the ear characterized by a purulent exudate and inflammation that can involve the ossicles, the auditory tube, and the mastoid bone. Test-Taking Tip: Make certain that the answer you select is reasonable and obtainable under ordinary circumstances and that the action can be carried out in the given situation. p. 384

Which is the most powerful hearing aid available for use by patients with hearing loss? A. In the ear B. In the canal C. Behind the ear D. Completely in the canal

C. Behind the ear Rationale A behind-the-ear hearing aid is the most powerful hearing aid and is used for all types of hearing loss. An in-the-ear hearing aid is used for mild to severe hearing loss, which provides powerful amplification. An in-the-canal hearing aid is also used for mild to severe hearing loss. A hearing aid set completely in the canal is used for mild to moderate hearing loss. p. 390

Which type of hearing loss occurs only when conditions in the outer or middle ear impair the transmission of sound through air to the inner ear? A. Mixed hearing loss B. Functional hearing loss C. Conductive hearing loss D. Sensorineural hearing loss

C. Conductive hearing loss Rationale Conductive hearing loss occurs when conditions in the outer or middle ear impair the transmission of sound through air to the inner ear. Mixed hearing loss occurs due to a combination of conductive and sensorineural causes. Functional hearing loss may be caused by an emotional or a psychologic factor. Sensorineural hearing loss is caused by impairment in function of the inner ear or the vestibulocochlear nerve (CN VIII). p. 387

What is the nurse's highest priority when caring for a patient experiencing an acute attack of vertigo? A. Educate the patient about diet restriction to prevent future episodes. B. Consult with an audiologist to arrange for an audiogram to be completed. C. Manage the patient's symptoms with antihistamines, anticholinergics, and benzodiazepines. D. Explain to the patient's spouse that a labyrinthectomy is the only successful treatment for vertigo.

C. Manage the patient's symptoms with antihistamines, anticholinergics, and benzodiazepines. Rationale Managing the patient's symptoms in acute attack of vertigo is the nursing Intervention that will bring the patient the most immediate relief, which makes it a top priority. Educating a patient about how to further prevent episodes of vertigo is not a priority in the acute management of vertigo; this can take place once the symptoms are under control. While an audiogram may help to confirm a diagnosis of Ménière disease, it is not necessary to provide an audiogram for the patient during an attack of vertigo. A labyrinthectomy is not the only recognized treatment for vertigo, nor is it a priority during the initial phase of the patient's illness. Test-Taking Tip: Identify option components as correct or incorrect. This may help you identify a wrong answer. p. 386

What substance is most appropriate for the nurse to use to remove an insect from a patient's ear? A. Water B. Alcohol C. Mineral oil D. Hydrogen peroxide

C. Mineral oil Rationale Mineral oil causes the least amount of trauma and irritation to the ear canal. Water and hydrogen peroxide should not be used because the insect could swell, which would make it more difficult to remove. Alcohol may cause both irritation of the ear canal and swelling of the insect. p. 384

Which auditory disorder is caused by an excessive accumulation of endolymph in the membranous labyrinth? A. Otitis Media B. Otosclerosis C. Ménière's disease D. Acoustic neuroma

C. Ménière's disease Rationale Ménière's disease, also called endolymphatic hydrops, results in an excessive accumulation of endolymph in the membranous labyrinth. The volume of endolymph increases until the membranous labyrinth ruptures. Otitis media is caused by an infection of the tympanum, ossicles, and space of the middle ear. Otosclerosis is a hereditary autosomal dominant disease caused by the vascular and bony changes in the middle ear. Otoscopic examination may reveal Schwartz's sign. Acoustic neuroma is a disorder of cranial nerve VIII, on which a unilateral benign tumor grows. p. 386

Which disorder can malignant external otitis cause in an older patient with diabetes mellitus? A. Otalgia B. Otosclerosis C. Osteomyelitis D. Acoustic neuroma

C. Osteomyelitis Rationale Malignant external otitis, a serious infection caused by the bacteria Pseudomonas aeruginosa, occurs mainly in older patients with diabetes mellitus. This infection can spread from the external ear to the parotid gland and temporal bone, causing osteomyelitis. Otalgia, or ear pain, is one of the first signs of external otitis. Otosclerosis is a hereditary autosomal dominant disease in which a spongy bone develops from the bony labyrinth, which prevents the movement of the footplate of the stapes in the oval window. An acoustic neuroma is the result of a unilateral benign tumor at the spot where the eighth cranial nerve enters the internal auditory canal. Test-Taking Tip: You have at least a 25 percent chance of selecting the correct response in multiple-choice items. If you are uncertain about a question, eliminate the choices that you believe are wrong and then call on your knowledge, skills, and abilities to choose from the remaining responses. p. 383

A patient tells the nurse that the right ear feels like it is plugged but does not hurt. This symptom is associated with which condition? A. Mastoiditis B. External otitis C. Otitis media with effusion D. Perforation of the tympanic membrane

C. Otitis media with effusion Rationale Otitis media with effusion is inflammation of the middle ear with fluid in the middle ear space. The eustachian tube, which normally allows air into the ear to equalize pressure, is blocked. Negative pressure develops, and fluid is pulled into the middle ear space. This effusion causes the patient to experience the feeling of fullness in the ear and diminished hearing. Mastoiditis is a complication of chronic otitis media in which recurrent ear infection spreads to the mastoid bone of the skull, resulting in hearing loss. External otitis involves inflammation or infection of the epithelium of the auricle and ear canal. Perforation of the tympanic membrane occurs with otitis media when pressure from the infection ruptures the eardrum and purulent fluid drains out of the ear canal. p. 384

A patient reports an inability to understand speech in spite of being able to hear sounds properly. The patient has a history of Paget's disease of the bone and is being treated with antibiotics. The nurse recognizes that the patient is most likely experiencing which type of hearing loss? A. Mixed hearing loss B. Conductive hearing loss C. Sensorineural hearing loss D. Central and functional hearing loss

C. Sensorineural hearing loss Rationale The patient is able to hear sound but is unable to understand speech. The patient also has a history of Paget's disease of the bone and is receiving antibiotic therapy. These factors lead to sensorineural hearing loss. Sensorineural hearing loss is caused by impairment of function of the inner ear or the vestibulocochlear nerve. Mixed hearing loss occurs due to a combination of conductive and sensorineural causes. In conductive hearing loss, the patient often speaks softly because he can hear his own voice quite loud. The patient hears better in a noisy environment. In central and functional hearing loss, the patient is unable to interpret sound, including speech. This is caused mainly due to a problem in the brain area controlling speech. p. 387

An otoscopic examination of a patient reveals a positive Schwartz's sign. A tuning fork test and an audiogram demonstrate air-bone gaps. Which statement describes the disorder these tests are used to diagnose? A. Free-floating debris in the semicircular canal causes vertigo with specific head movements. B. A unilateral benign tumor grows where the vestibulocochlear nerve enters the internal auditory canal. C. The spongy bone develops from the bony labyrinth, which prevents the movement of the footplate of the stapes in the oval window. D. The patient experiences significant disability because of sudden, severe attacks of vertigo with nausea, vomiting, sweating, and pallor.

C. The spongy bone develops from the bony labyrinth, which prevents the movement of the footplate of the stapes in the oval window. Rationale The patient has otosclerosis, which can be diagnosed with otoscopic examination revealing a positive Schwartz's sign along with tuning fork tests and an audiogram identifying air-bone gaps. In otosclerosis, spongy bone develops from the bony labyrinth, which prevents the movement of the footplate of the stapes in the oval window, thereby reducing the transmission of vibrations to the inner ear fluids. This results in conductive hearing loss. In benign paroxysmal positional vertigo (BPPV), free-floating debris in the semicircular canal causes vertigo with specific head movements, such as getting out of bed, rolling over in bed, and sitting up from lying down. An acoustic neuroma is a unilateral benign tumor that occurs where the vestibulocochlear nerve enters the internal auditory canal. The tumor can compress the trigeminal and facial nerves and arteries within the internal auditory canal. In Ménière's disease, the patient experiences significant disability because of sudden, severe attacks of vertigo with nausea, vomiting, sweating, and pallor. p. 385

The registered nurse is preparing to teach a group of nursing students about the use of verbal aids when communicating with hearing-impaired patients. Which information does the nurse include in the teaching plan? A. Refrain from having light behind the patient. B. Speak in a clear voice and shout at the patient. C. Use simple sentences and rephrase sentences if required. D. Maintain eye contact and draw the attention of the patient with hand movements.

C. Use simple sentences and rephrase sentences if required. Rationale The use of simple sentences is an example of a verbal aid that the nurse can use while communicating with a hearing-impaired patient. If required, the nurse should rephrase the sentence and use different words to help the patient understand. As a nonverbal aid, the nurse should avoid light behind the speaker, not the patient. As a verbal aid, the nurse should speak in a normal voice directly into the better ear. The nurse should not shout to make the patient understand. As a nonverbal aid, the nurse should maintain eye contact and draw attention of the patient with hand movements. p. 390

Which instruction should the nurse provide while teaching the patient and caregiver how to reduce the risk of external otitis? A. "Report if you notice earwax in the ear canal." B. "Dry your ears with cotton-tipped applicators." C. "Never use a hair dryer to speed up water evaporation from the ear." D. "Turn your head to each side for 30 seconds to drain water from the ear."

D. "Turn your head to each side for 30 seconds to drain water from the ear." Rationale Swimming and showering may alter the flora of the external canal because of chemicals and contaminated water, and thus cause external otitis. The nurse should instruct the patient to turn his or her head to each side for 30 seconds at a time to help water run out of the ears. This would help keep the ear dry. Earwax is normal, and it protects the ear canal. Chronic excessive cerumen should be reported if it impairs the patient's hearing. If water gets inside the ears, they should not be dried with cotton-tipped applicators. Instead, a hair dryer set to low and held at least six inches from the ear can speed water evaporation. pp. 383-384

The nurse is admitting a patient with glaucoma who states that he or she is allergic to all sulfa drugs. Which medication order should the nurse question and immediately report to the primary care provider? A. Carteolol B. Dipivefrin C. Carbachol D. Acetazolamide

D. Acetazolamide Rationale Acetazolamide is an antiglaucoma drug and a carbonic anhydrase inhibitor that may cause immunologically mediated reactions and result in sulfa-type allergic reactions in patients allergic to sulfa. Carteolol is a nonselective beta blocker and may cause blurred vision, photophobia, and bradycardia. Dipivefrin is sympathomimetic and may cause side effects such as ocular discomfort and redness. Carbachol is an antiglaucoma drug and a cholinergic agent that may cause transient ocular discomfort, headache, and blurred vision. p. 381

A patient is diagnosed with otitis media with effusion. The nurse expects that the patient will most likely exhibit what symptoms? A. Pain, fever, malaise, and reduced hearing B. Limited movement of the footplate of the stapes in the oval window. C. Episodic vertigo, tinnitus, fluctuating sensorineural hearing loss, and aural fullness D. Feeling of fullness of the ear, a plugged feeling or popping, and decreased hearing

D. Feeling of fullness of the ear, a plugged feeling or popping, and decreased hearing Rationale Otitis media with effusion is an inflammation of the middle ear with an accumulation of fluid in the middle ear space. The patient may complain of a feeling of fullness of the ear, a plugged feeling or popping, and decreased hearing. The fluid may be thin, mucoid, or purulent. Pain, fever, malaise, and reduced hearing are symptoms of acute otitis media. Episodic vertigo, tinnitus, fluctuating sensorineural hearing loss, and aural fullness are signs of Ménière's disease. Spongy bone developments from the bony labyrinth, preventing movement of the footplate of the stapes in the oval window, are signs of otosclerosis. p. 384

What is important for the nurse to include in the postoperative care of the patient following a stapedectomy to correct otosclerosis? Select all that apply. A. Check the gag reflex. B. Encourage independence. C. Avoid changing the cotton padding. D. Instruct patient to refrain from lifting or bending. E. Inform patient that nausea and dizziness may occur.

D. Instruct patient to refrain from lifting or bending. E. Inform patient that nausea and dizziness may occur. Rationale The patient may experience dizziness, nausea, and vomiting as a result of stimulation of the labyrinth during surgery. The patient should take care to avoid sudden movements that may bring on or exacerbate vertigo. Actions that increase inner ear pressure, such as coughing, sneezing, lifting, bending, and straining during bowel movements, should be avoided. Place a cotton ball in the ear canal, and cover the ear with a small dressing. It is not necessary to check a gag reflex. The patient will need assistance early postoperatively for safety reasons, so the nurse should not promote independence. The cotton padding may need to be changed if there is excess drainage. pp. 386-387

What behavior does the nurse assessing a patient with hearing loss expect to encounter? A. Aggressiveness B. Constant repetition C. Answering questions appropriately D. Irritability with others not speaking up

D. Irritability with others not speaking up Rationale The patient with hearing loss will become irritable at others who do not speak loudly enough for the patient to hear. Answering questions appropriately, aggressiveness, and constant repetition are not associated with hearing loss. p. 388

Which type of presbycusis is correctly documented with its cause, associated hearing change, and prognosis? A. Neural B. Sensory C. Cochlear D. Metabolic

D. Metabolic Rationale Metabolic presbycusis is caused by the atrophy of blood vessels in the wall of the cochlea with an interruption of essential nutrient supply. It is associated with uniform loss for all frequencies accompanied by recruitment. The patient responds well to hearing aids. Neural presbycusis is caused by degenerative changes in the cochlea and spinal ganglion. It is associated with loss of speech discrimination, and amplification alone is not sufficient to treat this type of hearing loss. Sensory presbycusis responds well to sound amplification; however, it is caused by atrophy of the auditory nerve and loss of sensory hair cells. Associated hearing change is a loss of high-pitched sounds. Cochlear presbycusis is caused by a stiffening of the basilar membrane, which interferes with sound transmission in the cochlea. The associated hearing loss increases from low to high frequencies, and speech discrimination is affected with higher-frequency losses. This type of hearing loss is helped by appropriate forms of amplification. p. 391

A nurse is comparing otosclerosis and otitis media with effusion. Which statement is correct regarding these two diseases? A. Otosclerosis can be caused by changes in air pressure, whereas otitis media with effusion follows chronic sinus infections. B. Otosclerosis is an inflammation of the middle ear, whereas otitis media with effusion is a hereditary recessive disease. C. Otosclerosis may be accompanied by a reddish blush of the tympanum, whereas otitis media with effusion is accompanied by purulent discharge from the ear. D. Otosclerosis is treated by oral administration of sodium fluoride, with vitamin D and calcium carbonate, whereas otitis media with effusion does not require antibiotic therapy.

D. Otosclerosis is treated by oral administration of sodium fluoride, with vitamin D and calcium carbonate, whereas otitis media with effusion does not require antibiotic therapy. Rationale The hearing loss associated with otosclerosis may be stabilized by the oral administration of sodium fluoride, with vitamin D and calcium carbonate. These medications slow bone resorption and encourage the calcification of bony lesions. Otitis media with effusion usually resolves without treatment but may recur. Chronic sinus infections can lead to otitis media with effusion; so can barotrauma caused by pressure change. Otosclerosis is not associated with a change in air pressure. Otosclerosis is a hereditary autosomal dominant disease, whereas otitis media with effusion is an inflammation of the middle ear with a collection of fluid in the middle ear space. In a patient with otosclerosis, an otoscopic examination may reveal a reddish blush of the tympanum known as Schwartz's sign; this is caused by the vascular and bony changes within the middle ear. In otitis media with effusion, the patient does not experience pain, fever, or discharge from the ear. pp. 384-385

Identify a causative factor associated with a sensorineural hearing loss. A. Otosclerosis B. Impacted cerumen C. Perforation of the tympanic membrane D. Ototoxicity from chemotherapeutic medications

D. Ototoxicity from chemotherapeutic medications Rationale A sensorineural hearing loss involves damage to the neural component of hearing. Within the inner ear there are sensory hair cells in the cochlea that pick up impulses and initiate nerve impulse to the brain via the acoustic branch of the eighth cranial nerve. Medications, including some chemotherapy, can damage this nerve pathway of hearing. Otosclerosis is a hereditary condition causing a conductive hearing loss because of bony growth that limits the stapes from vibrating in response to sound. Impacted cerumen (earwax) causes a conductive hearing loss because the sound vibrations are blocked by excessive earwax in the ear canal. A perforated tympanic membrane (ruptured eardrum) causes a conductive hearing loss because the tympanic membrane is no longer intact to adequately conduct sound vibrations. Test-Taking Tip: Have confidence in your initial response to an item because it more than likely is the correct answer. p. 387

Which nonverbal aids can the nurse adopt to effectively communicate with a patient with hearing loss? A. Refraining from touching the client B. Having a light source behind the speaker C. Maintaining equal distance from both ears D. Refraining from covering the mouth or face with hands

D. Refraining from covering the mouth or face with hands Rationale The nurse should not cover his or her mouth or face with the hands to communicate effectively with a patient with hearing loss. Using touch can be an effective strategy in this situation. The nurse should refrain from having light behind the speaker. The nurse should move close to the better ear while communicating with the patient with hearing loss. p. 390

The nurse recalls that presbycusis is hearing loss primarily caused by what? A. Traumatic injury B. Congenital defect C. Conductive defect D. Sensorineural loss

D. Sensorineural loss Rationale Presbycusis is due primarily to sensorineural hearing loss. It is a common type of hearing loss in adults older than 50 years. Traumatic injury and congenital and conductive defects are not associated with presbycusis. Also, conductive hearing loss is the result of repeated trauma such as exposure to loud noises or drug toxicity. p. 391

A patient with septic shock is receiving multiple medications. Which intravenous (IV) medication is most likely to cause a hearing loss? A. Aspirin B. Ampicillin C. Dopamine D. Vancomycin

D. Vancomycin Rationale The IV medication in use that is most likely to cause a hearing loss is vancomycin, because it is an ototoxic medication. For that reason, serum drug levels are monitored to maintain therapeutic levels and reduce the risk of ototoxicity. Neither dopamine nor ampicillin is likely to cause hearing loss. Aspirin also can cause hearing loss, but it is not administered IV. p. 387

Which finding related to primary open-angle glaucoma would the nurse expect to find when reviewing a patient's history and physical examination report? A. Absence of pain or pressure B. Blurred vision in the morning C. Seeing colored halos around lights D. Eye pain accompanied with nausea and vomiting

A. Absence of pain or pressure Rationale Primary open-angle glaucoma is typically symptom free, which explains why patients can have significant vision loss before a diagnosis is made unless regular eye examinations are being performed. Primary angle-closure glaucoma manifestations include sudden, excruciating pain in or around the eye, seeing colored halos around lights, and nausea and vomiting. p. 379

A patient presenting with an itching, burning sensation and redness in the eye is diagnosed with allergic conjunctivitis. What interventions are most important to ease the symptoms? Select all that apply. A. Administer artificial tears, as prescribed. B. Instruct the patient to wash hands regularly. C. Instruct the patient to avoid the allergen if it is known. D. Administer topical antihistamines and corticosteroids. E. Instruct the patient to use individual or disposable towels.

A. Administer artificial tears, as prescribed. C. Instruct the patient to avoid the allergen if it is known. D. Administer topical antihistamines and corticosteroids. Rationale Allergic conjunctivitis is caused by exposure to any allergen. If the allergen is known, the patient should be instructed to avoid the allergen as much as possible. Artificial tears can be used to dilute the allergen and wash it from the eye. Topical antihistamines and corticosteroids can be used to further decrease the symptoms. Hand washing and using individual or disposable towels are general measures to prevent the spread of infection, but they are not specific to allergic conjunctivitis. pp. 371-372

Identify risk factors associated with development of cataracts. Select all that apply. A. Advanced age B. History of diabetes mellitus C. Exposure to ultraviolet light D. Eating a diet high in lutein E. History of eye chronic open-angle glaucoma F. History of prolonged therapy with systemic corticosteroids

A. Advanced age B. History of diabetes mellitus C. Exposure to ultraviolet light F. History of prolonged therapy with systemic corticosteroids Rationale Risk factors for developing cataracts include advanced age, exposure to ultraviolet light, and conditions in which blood glucose levels are elevated, such as diabetes mellitus, or patients receiving long-term corticosteroid medications. These conditions alter metabolic processes and lead to the lens becoming cloudy and then opaque. A diet high in lutein is thought to decrease the risk of age-related macular degeneration. Patients with chronic glaucoma are not at higher risk for cataracts. However, some patients develop glaucoma after having cataract surgery or having an artificial lens implanted. pp. 373-374

A patient reports sudden, severe pain in the eye accompanied by nausea and vomiting. The assessment findings of the patient indicated optic nerve atrophy and peripheral visual field loss. Which drug will the nurse expect to be prescribed by the primary health care provider? A. Betaxolol B. Besifloxacin C. Tropicamide D. Ranibizumab

A. Betaxolol Rationale Glaucoma is a group of disorders characterized by increased intraocular pressure and optic nerve atrophy and peripheral visual field loss. The symptoms of glaucoma are pain in or around the eye, nausea, and vomiting. Betaxolol is an antiglaucoma drug that decreases intraocular pressure. Besifloxacin is an antibiotic that is used to treat acute bacterial conjunctivitis. Tropicamide is a cycloplegic that is used to produce pupillary dilation. Ranibizumab is a selective inhibitor of endothelial growth factor that is used to slow vision loss in age-related macular degeneration. Test-Taking Tip: Recall the symptoms of glaucoma and use critical thinking to answer the correct answer. p. 381

A patient, discharged after eye surgery, is told to avoid activities that will increase intraocular pressure. Which activities should the patient avoid? Select all that apply. A. Eating B. Lifting C. Coughing D. Bending over E. Breathing deeply

B. Lifting C. Coughing D. Bending over Rationale Activities such as coughing, bending over, and lifting increase the pressure within the eye. Eating and breathing deeply do not involve straining or lowering the head, so the pressure within the eye is not increased. p. 376

Which condition involves inflammation of the vitreous cavity? A. Uveitis B. Blepharitis C. Otitis media D. Endophthalmitis

D. Endophthalmitis Rationale Endophthalmitis is intraocular inflammation of the vitreous cavity. Uveitis is inflammation of the uvea. Inflammation of the margins of the eyelids is called blepharitis. Infection of the tympanum, ossicles, and space of the middle ear is called otitis media. Test-Taking Tip: Avoid looking for an answer pattern or code. There may be times when four or five consecutive questions have the same letter or number for the correct answer. p. 382

A patient is diagnosed with strabismus. The nurse expects the patient to exhibit which symptoms? A. Inability to accommodate for near objects B. Discomfort, pruritus, and redness in the eye C. Red, swollen, and acutely tender area in the lid margin D. Inability to focus two eyes simultaneously on the same object

D. Inability to focus two eyes simultaneously on the same object Rationale Strabismus is a condition affecting eye muscles so that the patient cannot consistently focus both eyes simultaneously on the same object. It is caused if the eye muscles are affected. Hyperopia is a condition in which the patient is unable to see near objects. Discomfort, pruritus, redness, and mucopurulent drainage in the eye are the symptoms of bacterial conjunctivitis. Red, swollen, circumscribed, and acutely tender areas in the lid margin are the symptoms of hordeolum. p. 373

The nurse is discussing glaucoma prevention with a 52-year-old African American patient. Which statement by the patient reflects a correct understanding of glaucoma prevention? A. "I will visit my eye doctor every one to two years." B. "I will wear protective sunglasses while outside." C. "I will take lutein and vitamin E supplements for eye health." D. "There is nothing that can be done to prevent vision loss from glaucoma."

A. "I will visit my eye doctor every one to two years." Rationale Loss of vision as a result of glaucoma is a preventable problem. Teach the patient and the caregiver about the risk of glaucoma and that it increases with age. Stress the importance of early detection and treatment in preventing visual impairment. A comprehensive ophthalmic examination is important in identifying persons with glaucoma or those at risk of developing glaucoma. The current recommendation is for an ophthalmologic examination every two to four years for persons between ages 40 and 64 years, and every one to two years for persons age 65 years or older. African Americans in every age category should have examinations more often because of the increased incidence and more aggressive course of glaucoma in these individuals. Wearing protective sunglasses while outside may help to reduce the development of cataracts, not glaucoma. Lutein and vitamin supplements may be helpful for preventing macular degeneration, not glaucoma. p. 380

A nurse provides education to a group of people about eye health care. Which statements by group indicate that the teaching has been understood? Select all that apply. A. "Wash hands regularly to prevent the spread of diseases." B. "Avoid removing contact lenses if there is redness and pain in the eye." C. "Wear sunglasses and ensure proper nutrition to prevent cataract development." D. "Wear eye protection during hazardous work activities to reduce the risk of eye injuries." E. "Regular eye checkups help in early detection of disease and prevent further loss of vision."

A. "Wash hands regularly to prevent the spread of diseases." C. "Wear sunglasses and ensure proper nutrition to prevent cataract development." D. "Wear eye protection during hazardous work activities to reduce the risk of eye injuries." E. "Regular eye checkups help in early detection of disease and prevent further loss of vision." Rationale Proper care of the eye plays a vital role in maintaining eye health. Regular hand washing helps to prevent the spread of disease from one eye to the other. Wearing sunglasses and eating a proper diet help to prevent cataract development and age-related diseases. Wearing eye protection during hazardous work helps to reduce the risk of eye injuries. Regular eye checkups help in the early detection of diseases and prevent further loss of vision. Contact lenses should be removed if there is redness and pain in the eye. p. 370

Which instruction should the nurse give the patient who is prescribed verteporfin for age-related macular degeneration? A. "You should avoid direct exposure to sunlight while on treatment." B. "You can wear clothes with short sleeves after receiving treatment." C. "You should avoid lutein-containing, green, leafy vegetables while on treatment." D. "You should consume vitamin E-containing foods but avoid vitamin C-containing foods after receiving treatment."

A. "You should avoid direct exposure to sunlight while on treatment." Rationale Age-related macular degeneration is an eye condition that leads to the deterioration of the macula, leading to loss of central vision. Verteporfin is a photosensitizing drug that becomes active when exposed to a low-level laser light wave or sunlight and may cause thermal burns. Therefore the nurse instructs the patient to avoid direct exposure to sunlight. The patient should cover the body completely, rather than wear short sleeves, because any exposure of the skin to sunlight could activate the medication in that area, resulting in a thermal burn. Lutein-containing, green, leafy vegetables do not react with verteporfin. Therefore there is no need of avoiding lutein-containing, green, leafy vegetables. Vitamins C and E are helpful in reducing the risk of age-related macular degeneration. Test-Taking Tip: Identify option components as correct or incorrect. This may help you identify a wrong answer. p. 379

The nurse is caring for a patient suspected of having age-related macular degeneration. What symptoms should the nurse document and report regarding this disorder? A. Blurred, darkened vision B. Itching, burning, and redness C. Sudden, excruciating pain in the eye D. Decreased vision, abnormal color perception

A. Blurred, darkened vision Rationale Age-related macular degeneration (AMD) is an eye condition that leads to the deterioration of the center of the retina, called the macula, leading to loss of central vision. The patient with AMD has blurred and darkened vision, scotomas, and metamorphopsia. The symptoms of cataract are decreased vision, abnormal color perception, and glare. The symptoms of glaucoma are sudden, excruciating pain in or around the eye. The symptoms of allergic conjunctivitis are itching, burning, and redness. p. 378

Which instruction is most appropriate for a patient using contact lenses who is diagnosed with bacterial conjunctivitis? A. Discard all opened or used lens care products. B. Disinfect contact lenses by soaking in a cleaning solution for 48 hours. C. Put all used cosmetics in a plastic bag for one week to kill any bacteria before reusing. D. Disinfect all lens care products with the prescribed antibiotic drops for one week after infection.

A. Discard all opened or used lens care products. Rationale The patient who wears contact lenses and develops infections should discard all opened or used lens care products and cosmetics to decrease the risk of reinfection from contaminated products. The risk of conjunctivitis is increased with not disinfecting lenses properly, wearing contact lenses too long, or using water or homemade solutions to store and clean lenses. pp. 372-373

The nurse is assessing a patient for esotropia. What sign observed by the nurse is clinically significant related to this disorder? A. Eye deviating in B. Eye deviating up C. Eye deviating out D. Eye deviating down

A. Eye deviating in Rationale Strabismus is a condition in which the patient cannot consistently focus two eyes simultaneously on the same object. The condition in which one eye deviates in is called esotropia. If the eye deviates up, it is called hypertropia. If the eye deviates out, it is called exotropia. If the eye deviates down, it is called hypotropia. p. 373

A patient informs the nurse that allergy symptoms are occurring in the eyes. What symptoms should the nurse ask the patient about when assessing for allergic conjunctivitis? A. Itching B. Photophobia C. Protruding eyeball D. Purulent discharge

A. Itching Rationale Allergic conjunctivitis occurs when the conjunctiva becomes swollen or inflamed due to reaction caused by an allergen. The defining symptom of allergic conjunctivitis is itching. Photophobia is the symptom of epidemic keratoconjunctivitis. Protruding eyeball is the symptom of exophthalmos. Purulent discharge is the symptom of corneal ulcer. p. 371

A patient is advised to take acetazolamide for chronic glaucoma. What nursing interventions are appropriate to perform when administering acetazolamide? Select all that apply. A. Monitor electrolyte levels. B. Ask if the patient is allergic to sulfa drugs. C. Avoid use if the patient has a history of asthma. D. Caution the patient about decreased visual acuity. E. Avoid use if the patient is on high-dose aspirin therapy.

A. Monitor electrolyte levels. B. Ask if the patient is allergic to sulfa drugs. E. Avoid use if the patient is on high-dose aspirin therapy. Rationale Acetazolamide is used to decrease production of aqueous humor. The drug may cause allergic reactions in patients sensitive to sulfa drugs; therefore, the nurse should ask the patient if he or she has a history of allergy to sulfa drugs. The drug has a diuretic effect and requires the nurse to monitor the patient's electrolyte levels. The drug should not be given to people on high-dose aspirin therapy because it can have adverse effects. Visual acuity is not affected with acetazolamide, and the drug is not contraindicated in people with asthma. STUDY TIP: Develop a realistic plan of study. Do not set rigid, unrealistic goals. p. 381

Which surgical procedure involves an incision in the tympanum to release the increased pressure and exudate from the middle ear? A. Myringotomy B. Stapedotomy C. Tympanoplasty D. Mastoidectomy

A. Myringotomy Rationale A myringotomy is a surgical procedure being performed on a patient with acute otitis media. It involves an incision in the tympanum to release the increased pressure and exudate from the middle ear. A stapedotomy is a microdrill or laser surgical treatment that involves opening the footplate as a part of collaborative care of otosclerosis. A tympanoplasty, also called a myringoplasty, involves reconstruction of the tympanic membrane and/or the ossicles of a patient with otitis media. A mastoidectomy is a surgery performed along with a tympanoplasty to remove the infected portions of the mastoid bone in a patient with chronic otitis media. p. 384

Which statement regarding auditory problems among different populations is correct? A. Native Americans have an increased incidence of otitis media. B. Asian Americans have a higher incidence of hearing impairment. C. African Americans have a higher incidence of macular degeneration. D. Individuals between 30 and 60 years of age show the symptom of otosclerosis.

A. Native Americans have an increased incidence of otitis media. Rationale Native Americans have an increased incidence of otitis media compared to whites. Whites have a higher incidence of hearing impairment when compared with Asian Americans. Whites have a higher incidence of macular degeneration than African Americans. Symptoms of endolymphatic hydrops usually begin between 30 and 60 years of age. Otosclerosis is the most common cause of hearing loss in young adults. p. .368

On a home visit to a patient who underwent cataract surgery, the nurse finds that the patient has intense pain in the operated eye. What should be the immediate nursing action? A. Notify the surgeon. B. Administer eyedrops. C. Administer analgesics. D. Apply a cold compress.

A. Notify the surgeon. Rationale In the postoperative period after a cataract surgery, the pain is usually mild. However, if the patient complains of intense pain, it should be immediately communicated to the surgeon because it may indicate hemorrhage, infection, or increased intraocular pressure and thus may need prompt intervention. Analgesics can be administered after receiving a surgeon's prescription. Applying a cold compress or administering eye drops may not decrease the pain. p. 375

The nurse is assessing four patients with visual problems. Which patient does the nurse determine is exhibiting signs of retinal detachment? A. Patient A: "Everything looks like a cobweb to me." B. Patient B: "I am unable to distinguish colors because they have too much glare." C. Patient C: "I am unable to tolerate light and have a sensation of a foreign body in my eyes." D. Patient D: "I have itching and burning sensations in my reddened eyes."

A. Patient A: "Everything looks like a cobweb to me." Rationale Retinal detachment is a separation of the sensory retina and the underlying pigment epithelium, with fluid accumulation between the two layers. Patients with a detaching retina describe symptoms that include a "cobweb," "hairnet," or ring in the field of vision. Therefore the nurse will expect that Patient A has retinal detachment. For Patient B, abnormal color perception and glares indicate cataracts. Redness, photophobia, and foreign body sensation indicates epidemic keratoconjunctivitis for Patient C. Itching, burning, redness, and tearing indicate allergic conjunctivitis in Patient D. Test-Taking Tip: Chart/exhibit items present a situation and ask a question. A variety of objective and subjective information is presented about the patient in formats such as the medical record (e.g., laboratory test results, results of diagnostic procedures, progress notes, health care provider orders, medication administration record, health history), physical assessment data, and assistant/patient interactions. After analyzing the information presented, the test taker answers the question. These questions usually reflect the analyzing level of cognitive thinking. pp. 376-377

The nurse would refrain from administering a prescribed dose of pilocarpine two drops to both eyes if it was documented that the patient has which comorbidity? A. Secondary glaucoma B. Macular degeneration C. Benign prostatic hypertrophy D. Chronic obstructive pulmonary disease (COPD)

A. Secondary glaucoma Rationale Contraindications to the use of pilocarpine include secondary glaucoma, acute iritis, and acute inflammation of the anterior segment of the eye. Benign prostatic hypertrophy, macular degeneration, and COPD are not contraindications to using this medication. p. 380

The adult child of a patient with macular degeneration asks the nurse how to avoid developing the condition. Which intervention should the nurse include in teaching? Select all that apply. A. Stop smoking or do not start. B. Avoid exposure to ultraviolet light. C. Wash hands before touching face or eyes. D. Wear eye protection while doing yard work. E. Eat green leafy vegetables such as spinach or kale daily.

A. Stop smoking or do not start. B. Avoid exposure to ultraviolet light. E. Eat green leafy vegetables such as spinach or kale daily. Rationale Besides aging and family history, risk factors for macular degeneration include cigarette smoking and long-term exposure to ultraviolet light. A diet rich in lutein, found in green leafy vegetables, may reduce the risk of macular degeneration. Hand hygiene prevents infection. Use of eye protection prevents injury. pp. 378-379

A patient is diagnosed with epidemic keratoconjunctivitis. How should the nurse help the patient relieve the eye infection? A. Suggest the use of ice packs and dark glasses. B. Discourage the use of mild topical corticosteroids. C. Advise to avoid the use of topical antibiotic ointments. D. Advise to avoid taking any treatment because the condition is self-limiting.

A. Suggest the use of ice packs and dark glasses. Rationale Epidemic keratoconjunctivitis is an ocular adenoviral disease. It is spread by direct contact, including sexual activity. Treatment involves the use of ice packs to reduce irritation. Dark glasses are used to reduce photophobia. Treatment should not be avoided in this condition. In severe cases, therapy can include mild topical corticosteroids and topical antibiotic ointment. p. 372

A patient presents with a sty in the left eye. Which nursing actions are appropriate to manage the patient's condition? Select all that apply. A. Teach the patient to perform lid scrubs daily. B. Prepare the patient for surgical removal of the sty. C. Administer appropriate antibiotic ointments or drops, as prescribed. D. Teach the patient to gently cleanse the lid margins with baby shampoo. E. Instruct the patient to apply warm, moist compresses at least four times a day.

A. Teach the patient to perform lid scrubs daily. C. Administer appropriate antibiotic ointments or drops, as prescribed. E. Instruct the patient to apply warm, moist compresses at least four times a day. Rationale A sty, also called a hordeolum, is caused by a Staphylococcus aureus infection of the sebaceous glands in the lid margin. The infection is manifested as a red, swollen, circumscribed, and acutely tender area. The patient should be instructed to apply warm, moist compresses at least four times a day to decrease the swelling and tenderness. Lid scrubs should be performed daily. Infection should be treated with appropriate antibiotic ointments or drops, as prescribed. Surgical intervention may not be required because a sty may subside with basic interventions. Cleansing with baby shampoo is not required because there are no secretions or crusting. p. 370

The nurse has completed patient teaching for a patient who had cataract surgery on the left eye. Which statement by the patient indicates a need for further teaching? A. "I might feel some scratchiness in my left eye." B. "I should notice an improvement in my vision in a few days." C. "I will call my health care provider if I notice white drainage or redness in my left eye." D. "I will call my health care provider if I notice white drainage or redness in my left eye."

B. "I should notice an improvement in my vision in a few days." Rationale The patient will notice an improvement in vision after surgery not in a few days. Cataract surgery typically results in little to no pain, but the patient may have some scratchiness in the operative eye. Mild analgesics are usually sufficient to relieve any discomfort, but if the pain is sudden or intense, the patient should notify the health care provider because this may indicate hemorrhage, infection, or increased intraocular pressure (IOP). The patient should be instructed to call the health care provider if redness or drainage occurs in the affected eye. These can be signs of infection. pp. 373-375

When teaching a patient about the pathophysiology related to open-angle glaucoma, which statement is most appropriate? A. "The retinal nerve is damaged by an abnormal increase in the production of aqueous humor." B. "There is decreased draining of aqueous humor in the eye, causing pressure damage to the optic nerve." C. "The lens enlarges with normal aging, pushing the iris forward, which then covers the outflow channels of the eye." D. "There is a decreased flow of aqueous humor into the anterior chamber by the lens of the eye blocking the papillary opening."

B. "There is decreased draining of aqueous humor in the eye, causing pressure damage to the optic nerve." Rationale With primary open-angle glaucoma, there is increased intraocular pressure because the aqueous humor cannot drain from the eye. This leads to damage to the optic nerve over time. Primary angle-closure glaucoma is caused by the lens bulging forward and blocking the flow of aqueous humor into the anterior chamber. p. 379

Which statement is most appropriate when teaching a patient about timolol eye drops in the treatment of glaucoma? A. "You may feel some palpitations after instilling these eye drops." B. "You may have a temporary headache after instilling these drops." C. "You should withhold this medication if your blood pressure becomes elevated." D. "You should keep your eyes closed for 15 minutes after instilling these eye drops."

B. "You may have a temporary headache after instilling these drops." Rationale It is common for patients to have a temporary headache when instilling eye drops. This should not cause concern to the patient. Because timolol is a β-blocker, heart rate may slow and blood pressure is more likely to decrease if absorbed systemically. Closing the eyes for 15 minutes after instilling the eye drops is not necessary. p. 381

Which decibel loss indicates moderately severe hearing impairment? A. 52 dB B. 60 dB C. 85 dB D. 110 db

B. 60 dB Rationale A decibel loss in the range of 56 to 70 dB indicates moderately severe hearing impairment. Hearing loss in the range of 41 to 55 dB indicates moderate impairment. Hearing loss in the range of 71 to 90 dB indicates severe impairment. Hearing loss over 90 dB indicates profound impairment, associated with congenital deafness. p. 388

The nurse is reviewing refractive errors of the eye. Which statement does the nurse identify as being true? A. Presbyopia occurs when the eyeball elongates. B. Astigmatism is caused by an irregular corneal curvature. C. Myopia is an inability to accommodate for near objects. D. Hyperopia is an inability to accommodate for objects at a distance.

B. Astigmatism is caused by an irregular corneal curvature. Rationale Astigmatism is caused by an irregular corneal curvature. Presbyopia is the loss of accommodation associated with age. As the eye ages, the lens becomes larger, firmer, and less elastic. Myopia (nearsightedness) is an inability to accommodate for objects at a distance. Hyperopia (farsightedness) is an inability to accommodate for near objects. Test-Taking Tip: Never leave a question unanswered. Even if answering is no more than an educated guess on your part, go ahead and mark an answer. You might be right, but if you leave it blank, you will certainly be wrong and lose precious points. p. 368

Which visual problem will the nurse suspect in the patient who has symptoms of itching, irritation, intolerance towards light, with crusts on the lid margins and lashes? A. Cataract B. Blepharitis C. Retinal detachment D. Allergic conjunctivitis

B. Blepharitis Rationale Blepharitis is a common chronic bilateral inflammation of the lid margins associated with crusts on the lid margins. Blepharitis is also associated with itching, irritation, and photophobia. Cataract is a clouding of the lens in the eye, which affects vision by abnormal color perception and glaring. Retinal detachment is a separation of the sensory retina and the underlying pigment epithelium. The symptoms of retinal detachment are photopsia and seeing a ring or cobwebs in the field of vision. Allergic conjunctivitis is caused by exposure to an allergen and is associated with itching and swelling. Test-Taking Tip: Read the question carefully before looking at the answers: (1) Determine what the question is really asking; look for key words; (2) Read each answer thoroughly and see if it completely covers the material asked by the question; (3) Narrow the choices by immediately eliminating answers you know are incorrect. pp. 370-371

A patient is having retinal hemorrhages, anoxic cotton-wool spots, and macular swelling in the eye. What should the nurse closely monitor that is a contributing factor to this disorder? A. Glucose level B. Blood pressure C. Intraocular pressure D. Thyroid hormone levels

B. Blood pressure Rationale Hypertensive retinopathy is caused by high blood pressure that creates blockages in retinal blood vessels. The eye examination of a patient with hypertensive retinopathy reveals retinal hemorrhages, anoxic cotton-wool spots, and macular swelling. If the eye examination shows capillary microaneurysms, retinal swelling, and hard exudates, then the nurse will suspect nonproliferative retinopathy and high blood glucose level. Exposure keratitis is seen in patients who cannot close their eyes adequately because of protruding eyeballs, which is caused by increased thyroid hormone. Angle-closure glaucoma occurs because of pupil dilation. When the pupil remains partially dilated long enough, it may result in increased intraocular pressure. Test-Taking Tip: You should recollect the signs and symptoms of hypertensive retinopathy to answer this question correctly. p. 376

Which part of the eye is inflamed in keratitis? A. Sclera B. Cornea C. Conjunctiva D. Eyelid margins

B. Cornea Rationale Keratitis is an infection or inflammation of the cornea. Scleritis involves inflammation of the sclera. Inflammation of the conjunctiva is a clinical manifestation of conjunctivitis. Blepharitis is associated with inflammation of the margins of both eyelids. Test-Taking Tip: Identify option components as correct or incorrect. This may help you identify a wrong answer. p. 372

The nurse is preparing a patient for penetrating keratoplasty. What disorder does the nurse determine the patient is being treated for? A. Retinopathy B. Corneal scars C. Chronic open-angle glaucoma D. Age-related macular degeneration

B. Corneal scars Rationale A corneal scar is the chronic inflammation of the corneal stroma. Penetrating keratoplasty is the procedure performed to treat corneal scars. While performing surgery, the ophthalmic surgeon removes the full thickness of the patient's cornea and replaces it with a donor cornea that is sutured into place. Retinopathy is treated by laser photocoagulation. Chronic open-angle glaucoma is treated by argon laser trabeculoplasty. Age-related macular degeneration is treated by photodynamic therapy. p. 373

An asthmatic patient is diagnosed with chronic glaucoma. The patient is prescribed timolol (Istalol). In regard to patient safety, what action should the nurse take? A. Explain to patient that carteolol may cause vomiting. B. Do not administer istalol, and notify the health care provider. C. Ask the health care provider to decrease the dose of carteolol. D. Suggest the patient only use carteolol for a short period of time.

B. Do not administer istalol, and notify the health care provider. Rationale In an asthmatic patient, administration of timolol should be avoided because it causes bronchospasm. Decreasing the dose and suggesting that the patient only use the medicine for a short period of time do not reduce the risk of bronchospasm. Vomiting is not a side effect of timolol. p. 381

What instructions would the nurse teach a patient and caregiver regarding the prevention of external otitis? Select all that apply. A. Report any decrease in cerumen production. B. Do not dry the ears with a cotton-tipped applicator. C. Report itching of the cochlea if it becomes a problem. D. Use earplugs if the patient is prone to swimmer's ear. E. Do not put anything in the ear canal unless instructed by the healthcare provider.

B. Do not dry the ears with a cotton-tipped applicator. D. Use earplugs if the patient is prone to swimmer's ear. E. Do not put anything in the ear canal unless instructed by the healthcare provider. Rationale Wearing earplugs can prevent contaminated water from contacting the external canal, thereby preventing the alteration of normal flora leading to bacterial growth. Ears should be cleaned and dried with a washcloth, as cotton-tipped applicators can cause serious injury to the tympanic membrane. Putting objects into the ear canal can cause impaction of cerumen, impairing hearing, or penetration of the middle ear. An increase, not decrease, in cerumen production has been shown to have an effect on development of external otitis. Itching ears are a common symptom with external otitis; however, the cochlea is in the inner ear and does not itch. pp. 383-384

When planning care for a patient with disturbed sensory perception related to increased intraocular pressure caused by primary open-angle glaucoma, on what should the nurse focus? A. Giving anticipatory guidance about the eventual loss of central vision that will occur B. Encouraging compliance with drug therapy for the glaucoma to prevent loss of vision C. Recognizing that eye damage caused by glaucoma can be reversed in the early stages D. Managing the pain experienced by patients with glaucoma that persists until the optic nerve atrophies

B. Encouraging compliance with drug therapy for the glaucoma to prevent loss of vision Rationale Drug therapy is necessary to prevent the eventual vision loss that may occur as a consequence of glaucoma. For this reason, the patient should be encouraged to remain compliant with drug therapy. Glaucoma cannot be reversed. Central vision usually is unaffected with open-angle glaucoma. Pain management is important throughout the course of the disease. p. 380

A nurse administered tropicamide in both eyes of a patient in the preoperative room before cataract surgery. What is the most important nursing intervention for this patient? A. Brighten the room with extra lights. B. Instruct the patient to wear dark glasses. C. Monitor for pulmonary effects of the drug. D. Reassure the patient that the surgery will be uneventful.

B. Instruct the patient to wear dark glasses. Rationale Tropicamide is a cycloplegic. It dilates the pupil by blocking the effect of acetylcholine on the iris sphincter muscle. It causes photophobia, so the nurse should instruct the patient to wear dark glasses. The nurse should also dim the lights in the room. The drug may also be absorbed systemically; therefore, the nurse should observe the patient for tachycardia and other effects on the central nervous system. Tropicamide does not cause pulmonary side effects. STUDY TIP: Enhance your time-management abilities by designing a study program that best suits your needs and current daily routines by considering issues such as the following: (1) Amount of time needed; (2) Amount of time available; (3) "Best" time to study; (4) Time for emergencies and relaxation. p. 374

A patient reports an inability to see near objects. The nurse recalls that which physiologic condition is responsible for this disorder? A. Cornea having irregular curvature B. Light rays focusing behind the retina C. Incoming light rays bending unequally D. Light rays focusing in front of the retina

B. Light rays focusing behind the retina Rationale The patient with an inability to see near objects suffers from hyperopia or farsightedness. In this case, the light rays focus behind the retina. Normally the light rays should focus on the retina for near objects. This type of refractive error occurs when the cornea or lens does not have adequate focusing power or when the eyeball is too short. Irregular corneal curvature and incoming light rays bending unequally are associated with astigmatism. Focusing light rays in front of the retina is the sign of myopia. p. 368

In presbyopia the lens of the eye loses flexibility and is unable to accommodate close vision. The nurse recognizes that this condition generally occurs in which group? A. Adolescents and young adults B. Men and women older than 40 C. Men between the ages of 30 and 50 D. Women between the ages of 20 and 40

B. Men and women older than 40 Rationale Presbyopia is an age-related change in vision that generally occurs in men and women older than 40 years. Adolescents and young adults are not subject to the condition, and it does not affect one gender exclusively. p. 368

The nurse is assessing four patients with different refractive errors. Which patient will the nurse suspect to have hyperopia? A. Patient A: Inability to accommodate for objects at a distance B. Patient B: Inability to accommodate for near objects C. Patient C: Loss of accommodation which is associated to age D. Patient D: Irregular corneal curvature

B. Patient B: Inability to accommodate for near objects Rationale Hyperopia, or farsightedness, is an inability to accommodate near objects. It causes the light rays to focus behind the retina and requires the patient to use accommodation to focus the light rays on the retina for near objects; therefore, patient B is suspected to have hyperopia. Myopia or nearsightedness is an inability to accommodate objects at a distance; therefore, Patient A has myopia. Presbyopia is the loss of accommodation associated with age; therefore, Patient C has presbyopia. Astigmatism is caused by an irregular corneal curvature; therefore, Patient D has astigmatism. p. 368

A patient has severe myopia. Which type of correction is the patient planning to have if the patient tells the nurse, "I can't wait to be able to see after they implant a contact lens over my lens"? A. Photorefractive keratectomy (PRK) B. Phakic intraocular lenses (phakic IOLs) C. Refractive intraocular lens (refractive IOL) D. Laser-assisted in situ keratomileusis (LASIK)

B. Phakic intraocular lenses (phakic IOLs) Rationale Phakic IOL is the implantation of a contact lens in front of the natural lens. PRK is used with low to moderate amounts of myopia; the epithelium is removed, and the laser sculpts the cornea to correct the refractive error. Refractive IOL also is for patients with a high degree of myopia or hyperopia and involves removing the natural lens and implanting an intraocular lens. LASIK surgery is similar to PRK except that the epithelium is replaced after surgery. p. 368

Which surgery treats age-related macular degeneration (AMD) by destroying abnormal blood vessels without causing permanent damage to the retinal pigment epithelium and photoreceptor cells? A. Filtration surgery B. Photodynamic therapy C. Laser photocoagulation D. Argon laser trabeculoplasty

B. Photodynamic therapy Rationale Photodynamic therapy treats AMD by destroying abnormal blood vessels without causing permanent damage to the retinal pigment epithelium and photoreceptor cells. Filtration surgery is the treatment for chronic open-angle glaucoma but not for AMD. Laser photocoagulation is used for treatment of proliferative retinopathy. Argon laser trabeculoplasty is a noninvasive procedure to lower the intraocular pressure in glaucoma. pp. 378-379

While at work a patient has a penetrating eye injury from a foreign object. What action should the occupational health nurse take while awaiting arrival of the emergency response team? A. Place the patient in a flat supine position. B. Stabilize the foreign object within the injury site. C. Instruct the patient to bend over and take deep breaths. D. Continuously irrigate the eye with sterile saline solution.

B. Stabilize the foreign object within the injury site. Rationale The nurse should stabilize the foreign object penetrating the eye to prevent further damage until the injury can be treated by an emergency medicine specialist and ophthalmologist. The head of the patient should be elevated 45 degrees to prevent excessive pressure within the eye. Bending over is avoided as well. Irrigation of the eye is indicated only if the injury is caused by a chemical exposure. Test-Taking Tip: Answer the question that is asked. Read the situation and the question carefully, looking for key words or phrases. Do not read anything into the question or apply what you did in a similar situation during one of your clinical experiences. Think of each question as being an ideal, yet realistic, situation. p. 371

A patient is using dipivefrin for the treatment of glaucoma. What side effects should the nurse educate the patient to monitor? Select all that apply. A. Depression B. Tachycardia C. Hypertension D. Bronchospasm E. Taste alteration

B. Tachycardia C. Hypertension Rationale Dipivefrin is an α-adrenergic agonist and is converted to epinephrine inside the eye. It decreases aqueous humor production and enhances outflow facility. Dipivefrin causes tachycardia and hypertension. Depression and bronchospasm are side effects of the antiglaucoma drug carteolol. Taste alteration is a side effect of carbonic anhydrase inhibitors. Test-Taking Tip: Be alert for details about what you are being asked to do. In this question type, you are asked to select all options that apply to a given situation or patient. All options likely relate to the situation, but only some of the options may relate directly to the situation. p. 381

A patient has undergone cataract surgery. What nursing interventions help to prevent postoperative complications? Select all that apply. A. Antiviral medications are given to prevent infections. B. Teach the patient to instill medicine following aseptic techniques. C. Teach the patient about proper hygiene and eye care techniques. D. Ask the patient to discontinue all prescribed medicine two days after surgery. E. Advise the patient to avoid actions that can cause increased intraocular pressure.

B. Teach the patient to instill medicine following aseptic techniques. C. Teach the patient about proper hygiene and eye care techniques. E. Advise the patient to avoid actions that can cause increased intraocular pressure. Rationale Postoperative care of the eye is essential for proper wound healing. Increased intraocular pressure may break the sutures and increases the risk of complications; therefore, it should be prevented. Proper hygiene and eye care techniques prevent contamination of the wound. Medicines should be instilled in the eye using aseptic techniques to prevent infection. The full course of medicine should be completed to obtain the therapeutic effect. Antibiotic drops, not antiviral medications, are given to prevent infections. pp. 373-375

Which statement by the student nurse indicates the need for further teaching regarding age-related macular degeneration (AMD)? A. "AMD is related to retinal aging." B. "Family history is a major risk factor for AMD." C. "People with dark-colored eyes are more at the risk for AMD." D. "Long-term exposure to ultraviolet light is a risk factor for AMD."

C. "People with dark-colored eyes are more at the risk for AMD." Rationale Age-related macular degeneration is the most common cause of irreversible central vision loss in people above 60 years of age. People with light-colored eyes, not dark-colored eyes, are more at risk for AMD because light eyes have less pigment, which makes them sensitive to light, causing AMD. The student making this statement requires further teaching. AMD is related to retinal aging because changes in astrocytes in retinal aging cause retinal ischemia, which leads to AMD. Genetic factors play a major role in AMD, and family history is a major risk factor for AMD because multiple genetic variants are involved in AMD. Long-term exposure to ultraviolet light is a risk factor for AMD because long-term exposure may cause retinal detachment. p. 378

A patient has experienced a sudden decrease in vision. During an eye examination, the patient overhears the primary health care provider mention that the patient has papilledema, and asks the nurse to explain what that is. Which answer by the nurse is correct? A. "Papilledema is caused by irritants and microorganisms." B. "Papilledema is fluid accumulation between two layers within the retina." C. "Sustained, severe high blood pressure can cause swelling of the optic disc and nerve, resulting in papilledema." D. "This condition is caused by the development of abnormal blood vessels in or near the macula inside your eye."

C. "Sustained, severe high blood pressure can cause swelling of the optic disc and nerve, resulting in papilledema." Rationale Sustained, severe hypertension can cause sudden visual loss from swelling of the optic disc and nerve. This condition is known as papilledema. It is not caused by the development of abnormal blood vessels in or around the macula, nor is it caused by microorganisms. Papilledema is not fluid accumulation between two layers within the retina. The development of abnormal blood vessels in or near the macula is known as macular degeneration. p. 376

The nurse assesses a patient with a red, swollen, circumscribed, acutely tender area near the eye and in the lid margin. What action does the nurse anticipate providing to assist with the relief of discomfort due to this condition? A. Administration of artificial tears B. Insertion of intacs on the cornea C. Application of a warm, moist compress four times a day D. Administration of nonsteroidal antiinflammatory eye drops

C. Application of a warm, moist compress four times a day Rationale Hordeolum is an infection of the sebaceous glands in the lid margin. The symptoms of hordeolum are a red, swollen, circumscribed, and acutely tender area near the eye. The only treatment that may be necessary for this patient is to apply warm, moist compresses four times a day, which will decrease the swelling and redness of the eye. Administering artificial tears is the treatment for allergic conjunctivitis to dilute the allergen and wash it away. Insertion of intacs on the cornea is the treatment for keratoconus. Administration of nonsteroidal antiinflammatory eye drops is the treatment given if the patient has inflammation. p. 370

While evaluating a patient the nurse suspects primary open-angle glaucoma if which classic symptom is present? A. Vacillating pupil B. Constant tearing C. Decreased peripheral vision D. Colored halos around lights

C. Decreased peripheral vision Rationale Primary open-angle glaucoma (POAG) develops slowly and without symptoms. The gradual loss of peripheral vision is one of the diagnostic criteria for primary open-angle glaucoma, which manifests as tunnel vision late in POAG. Vacillating pupils and constant tearing are not directly associated with any form of glaucoma. Colored halos around lights are seen in acute-angle closure glaucoma, which is less common than POAG. Acute-angle closure glaucoma is an ocular emergency requiring immediate intervention because intraocular pressure increases rapidly and may cause optic nerve damage and blindness. p. 378

A patient is discharged to home after cataract surgery. What is the most important instruction the nurse should include in the discharge teaching? A. Restrict activity at home. B. Wear a nighttime eye shield. C. Do not bend, stoop, cough, or lift. D. Wash hands before touching the eye.

C. Do not bend, stoop, cough, or lift. Rationale When teaching postoperative home care for cataract surgery, the nurse should instruct the patient to avoid activities such as bending, stooping, coughing, or lifting because these activities can raise the intraocular pressure, which in turn can adversely affect the newly implanted lens. All other activities are permissible. Wearing a nighttime eye shield is not necessary. Washing hands before touching the eye is a general hygiene practice and not specific to postoperative care. pp. 374-375

A 68-year-old patient has undergone a total hip replacement and has glaucoma. The nurse forms a nursing diagnosis of disturbed sensory perception related to increased intraocular pressure. The plan of care should focus on which main element? A. Restriction of driving privileges immediately B. Use of occupational and physical therapy for visual deficits C. Encouraging medication compliance to reduce the risk of vision loss D. Managing the pain using oral antiinflammatories and opioids as needed

C. Encouraging medication compliance to reduce the risk of vision loss Rationale Drug therapy is necessary to prevent the eventual vision loss that accompanies glaucoma. For this reason, the nurse should encourage the patient to remain compliant with drug therapy. Physical therapy will not improve or treat visual deficits. Glaucoma does not cause pain, and unless the vision is severely impaired, driving restrictions are not necessary. pp. 380, 382

A nurse should instruct a patient who had cataract surgery to contact the surgeon if which condition develops? A. Glare B. Itching C. Eye pain D. Blurred vision

C. Eye pain Rationale Pain should not be present after cataract surgery, although there may be slight discomfort that is easily relieved with acetaminophen. The patient should be told that the other symptoms, including glare, itching, and blurred vision, may be present and are expected until healing takes place. p. 375

A patient asks the nurse, "How does glaucoma damage my eyesight?" What explanation should the nurse provide to the patient? A. Glaucoma leads to detachment of the retina. B. Glaucoma results from chronic eye inflammation. C. Glaucoma results in increased intraocular pressure. D. Glaucoma is caused by decreased blood flow to the retina.

C. Glaucoma results in increased intraocular pressure. Rationale In chronic open-angle glaucoma the outflow of aqueous humor is obstructed, leading to increased intraocular pressure. The increased intraocular pressure eventually causes destruction of the nerve fibers of the retina and painless vision loss, beginning in the periphery. Glaucoma does not cause detachment of the retina, result from chronic inflammation, or result from decreased retinal blood flow. p. 379

When administering eye drops to a patient with glaucoma, which nursing measure is most appropriate to minimize systemic effects of the medication? A. Apply pressure to each eyeball for a few seconds after administration. B. Have the patient close the eyes and move them back and forth several times. C. Have the patient put pressure on the inner canthus of the eye after administration. D. Have the patient try to blink out excess medication immediately after administration.

C. Have the patient put pressure on the inner canthus of the eye after administration. Rationale Systemic absorption can be minimized by applying pressure to the inner canthus of the eye. Applying pressure to each eyeball, having the patient close the eyes and move them back and forth, and having the patient try to blink out excess medication will not minimize systemic effects of the medication. p. 382

A patient is diagnosed with proliferative retinopathy and is scheduled for treatment by the primary care provider. On which treatment option does the nurse educate the patient? A. Filtration surgery B. Photodynamic therapy C. Laser photocoagulation D. Argon laser trabeculoplasty

C. Laser photocoagulation Rationale Proliferative retinopathy is a condition associated with the formation of fragile new abnormal blood vessels, which are predisposed to leaks, resulting in severe vision loss. This condition can be treated by laser photocoagulation. Filtration surgery is the treatment for chronic open-angle glaucoma. Photodynamic therapy is the treatment for age-related macular degeneration. Argon laser trabeculoplasty is a noninvasive procedure to lower intraocular pressure in glaucoma. Test-Taking Tip: You have at least a 25 percent chance of selecting the correct response in multiple-choice items. If you are uncertain about a question, eliminate the choices that you believe are wrong and then call on your knowledge, skills, and abilities to choose from the remaining responses. p. 377

Which symptom occurs initially with retinal detachment? A. Redness of the conjunctiva B. Increased glare with artificial light C. Seeing flashes of light and floaters D. Severe pain when moving the eyes

C. Seeing flashes of light and floaters Rationale A detached retina involves the separation of the sensory retina from the underlying pigment epithelium. Fluid accumulates in the space and blocks essential nourishment and circulation to retinal cells. Initially symptoms include seeing flashes of light, an increased amount of floaters, or lines in the vision field. If the detachment advances, there is a loss of vision peripherally or centrally, depending where the detachment has occurred. Eye irritation or infection causes redness of the conjunctiva. Increased glare that interferes with visual acuity is noted by patients with cataracts. Inflammation or infection within the eye causes pain with eye movement. pp. 376-377

A nurse reviews the medical record for a patient with acute glaucoma for which acetazolamide has been prescribed. The patient has a history of high-dose aspirin therapy. Considering the concomitant use of the medications, the nurse expects what change in medication prescriptions? A. The dose of acetazolamide will be decreased. B. There will be no change in prescriptions of either medication. C. The patient cannot take both medications due to gastric disturbances. D. The patient will be advised to take acetazolamide at a different time than aspirin.

C. The patient cannot take both medications due to gastric disturbances Rationale Acetazolamide is a carbonic anhydrase inhibitor used in the treatment of glaucoma. The patient is on high-dose aspirin therapy; therefore, acetazolamide is avoided because it increases the risk of gastric disturbances. Decreasing the dose of acetazolamide may not decrease the risk of developing complications. A combination of acetazolamide and aspirin is not recommended because it can cause GI upset. Taking acetazolamide at a different time than aspirin does not prevent drug interactions and should be avoided. p. 381

A patient has sustained an injury to the cornea. The nurse assists the primary health care provider in rinsing the eye with saline solution and then instilling a dye into the conjunctiva of the injured eye. What is the purpose of the dye? A. To disinfect the injured tissue B. To help seal and heal the injured tissue C. To stain the injured tissue so it can more easily be identified D. To bind with foreign particles, allowing them to be rinsed away from injured tissue

C. To stain the injured tissue so it can more easily be identified Rationale The injured tissue is susceptible to the dye and will remain stained even after the eye is rinsed with saline. Fluorescein is an ophthalmic diagnostic dye used to identify corneal defects and locate foreign objects in the eye. The dye is not used to disinfect the tissue, seal and heal it, or bind with foreign particles. p. 378

The nurse is caring for a patient with keratitis caused by the herpes simplex virus. What order should the nurse question prior to administering? A. Oral acyclovir B. Trifluridine drops C. Topical corticosteroids D. Topical vidarabine ointment

C. Topical corticosteroids Rationale Keratitis is an inflammation or infection of the cornea that can be caused by a variety of microorganisms. Topical corticosteroids are contraindicated in this patient because they may cause deeper ulceration of the cornea on prolonged treatment. Oral acyclovir is an antiviral medication that is effective in the treatment of viral keratitis. Trifluridine drops and topical vidarabine ointment are effective in treating viral keratitis. p. 372

A patient left blind as a result of a motor vehicle accident is withdrawn and refuses to get out of bed. What is the nurse's priority goal for this patient? A. Use suitable coping strategies to reduce stress B. Identify patient's strengths and support system C. Verbalize feelings related to visual impairment D. Transition successfully to the sudden vision loss

C. Verbalize feelings related to visual impairment Rationale The nurse's priority is to help the patient express feelings about vision loss because the patient is not coping effectively with the situation. Until the patient expresses how he or she feels, the patient will be unable to progress in the rehabilitation process, including using suitable coping strategies to reduce stress, developing strengths and a support system, and transitioning successfully to the sudden vision loss. p. 382

A patient has lost an eye after an industrial accident. Which action by the nurse is most appropriate during this time? A. Speak louder when talking to the patient. B. Avoid making eye contact during a conversation. C. Introduce the patient to other visually impaired persons. D. Assist the patient with the same grieving process that is associated with other losses.

D. Assist the patient with the same grieving process that is associated with other losses. Rationale When the patient has lost visual function or even the entire eye, he or she will grieve the loss. The nurse should help the patient through the grieving process. The patient lost an eye, not an ear, so speaking louder is not necessary. The nurse should still make eye contact with the patient. Introducing the patient to other visually impaired persons is not recommended early in the grieving process. STUDY TIP: A word of warning: do not expect to achieve the maximum benefits of this review tool by cramming a few days before the examination. It doesn't work! Instead, organize planned study sessions in an environment that you find relaxing, free of stress, and supportive of the learning process. p. 382

Before administrating timolol eye drops for treatment of glaucoma, the nurse would assess the patient for which contraindication for the use of this medicine?A. Sinusitis B. Migraine headaches C. Chronic urinary tract infection D. Chronic obstructive pulmonary disease (COPD)

D. Chronic obstructive pulmonary disease (COPD) Rationale Timolol is a nonselective β-adrenergic blocker that could lead to bronchoconstriction and bronchospasm. For this reason, it should not be used in patients with COPD. Timolol may be used to treat migraine headaches and does not affect sinusitis or chronic urinary tract infections. p. 381

A patient admitted for a total knee replacement has a history of primary open-angle glaucoma. The nurse expects to see which finding recorded in the history and physical examination report? A. Diplopia B. Frequent falls C. Decreased visual acuity D. Denial of pain or pressure

D. Denial of pain or pressure Rationale Primary open-angle glaucoma is typically symptom-free, which explains why patients can have significant vision loss before diagnosis unless regular eye examinations are performed. Glaucoma does not result in diplopia or frequent falls. p. 379

A patient is diagnosed with bacterial conjunctivitis. The nurse expects what patient symptoms? A. Itching, burning, irritation, and photophobia B. Tearing, redness, photophobia, and foreign body sensation C. Red, swollen, circumscribed, and acutely tender area in the lid margin D. Discomfort, pruritus, redness, and a mucopurulent drainage in the eye

D. Discomfort, pruritus, redness, and a mucopurulent drainage in the eye Rationale Bacterial conjunctivitis manifests as discomfort, pruritus, redness, and a mucopurulent drainage in the eye. It occurs due to unhygienic conditions. The infection is caused by Staphylococcus aureus. It can be treated with antibiotic drops. Itching, burning, irritation, and photophobia are signs of blepharitis. Tearing, redness, photophobia, and foreign body sensation are symptoms of epidemic keratoconjunctivitis. Red, swollen, circumscribed, and acutely tender areas in the lid margin are the symptoms of hordeolum. p. 371

A patient has undergone kidney transplantation surgery and takes immunosuppressant drugs. The patient comes in contact with a person who has chickenpox. The nurse recognizes that the patient is susceptible to which viral ophthalmic infection? A. Blepharitis B. Hordeolum C. Astigmatism D. Herpes zoster ophthalmicus

D. Herpes zoster ophthalmicus Rationale The patient is being treated with immunosuppressant drugs to prevent the rejection of a transplanted kidney. The immunosuppressive drugs tend to decrease the patient's immunity. If the patient comes in contact with a patient with chickenpox, there is the possibility of herpes zoster ophthalmicus infection. It may occur due to reactivation of an endogenous infection. The endogenous infection might have persisted in a latent form after an earlier attack of varicella or by contact with a patient with chickenpox or herpes zoster. It occurs most frequently in older adults and immunosuppressed patients. Blepharitis is a common chronic bilateral inflammation of the lid margins. Hordeolum is an infection of the sebaceous glands in the lid margin. Astigmatism is an eye disorder; it is not a viral infection. p. 372

An elderly patient with a history of bilateral cataracts is admitted to the hospital with pneumonia. What intervention will facilitate the patient's ability to see? Select all that apply. A. Administer prescribed analgesics. B. Patch the eye that has less visual acuity. C. Obtain dark glasses for the patient to wear. D. Increase the amount of light for near vision. E. Obtain teaching materials with enlarged print.

D. Increase the amount of light for near vision. E. Obtain teaching materials with enlarged print. Rationale The patient with intact cataracts will see better with the use of increased lighting and magnifiers, including enlarged print. Receiving prescribed analgesics will relieve discomfort. Patching an eye will lessen vision. Wearing dark glasses decreases the patient's visual acuity. pp. 373-374

A patient with wet age-related macular degeneration (AMD) has undergone phototherapy with intravenous verteporfin and a cold laser. What is the most important nursing intervention for this patient? A. Instruct the patient to quit smoking. B. Suggest that the patient consider using supplements of vitamins and minerals. C. Advise the patient to eat lots of dark green, leafy vegetables containing lutein. D. Instruct the patient to avoid direct exposure to sunlight for five days after treatment.

D. Instruct the patient to avoid direct exposure to sunlight for five days after treatment. Rationale Verteporfin, used for phototherapy, is a photosensitizing drug. It becomes active in the presence of low-level laser light waves. It can be activated by exposure to sunlight or high-intensity light until it is fully excreted. If activated, it can cause thermal burns in the area. The patient should be instructed to avoid direct sunlight and other intense forms of light for five days after treatment. Taking vitamin and mineral supplements, eating green leafy vegetables high in lutein, and cessation of smoking are measures to decrease the risk of AMD. pp. 378-379

An older adult patient tells the nurse, "I feel like there is sand in my eye." Which condition will the nurse suspect? A. Cataract B. Strabismus C. Keratoconus D. Keratoconjunctivitis sicca

D. Keratoconjunctivitis sicca Rationale Keratoconjunctivitis sicca is a dry eye disorder commonly seen in older adults. The patient with keratoconjunctivitis sicca reports irritation and presence of sand in the eye. Cataract is a condition in which there is opacity within the lens and the patient reports decreased vision, abnormal color perception, and glare. Strabismus is a condition in which the patient cannot consistently focus two eyes simultaneously on the same object and the patient will complain of double vision. Keratoconus is a noninflammatory disorder in which the anterior cornea thins and protrudes forward, taking on a cone shape, and the patient complains of blurred vision. p. 373

While assessing a patient with systemic lupus erythematosus (SLE), the nurse observes the patient rubbing the eyes frequently and decreased tear production. What condition should the nurse educate the patient regarding? A. Cataract B. Strabismus C. Keratoconus D. Keratoconjunctivitis sicca

D. Keratoconjunctivitis sicca Rationale Keratoconjunctivitis sicca is a dry eye disorder particularly of older adults and individuals with certain systemic diseases such as scleroderma and systemic lupus erythematosus. The patient has decreased tear secretion because of a decrease in the quality or quantity of the tear film. Cataract is a clouding of the lens in the eye, which affects the vision. The symptoms of cataract are decreased vision, abnormal color perception, and glare. Strabismus is a condition in which the patient cannot consistently focus two eyes simultaneously on the same object and has double vision. Keratoconus is a noninflammatory eye disorder in which the anterior cornea thins and protrudes forward, taking on a cone shape and resulting in blurred vision. Test-Taking Tip: You should recollect the relation with systemic lupus erythematosus and eye disorder to answer this question accurately. p. 373

When performing an assessment for a patient with glaucoma, the nurse observes brown iris pigmentation. Which antiglaucoma drug does the nurse determine the patient is taking? A. Carteolol B. Dipivefrin C. Carbachol D. Latanoprost

D. Latanoprost Rationale Latanoprost is an antiglaucoma drug that stimulates melanin production in melanocytes and increases the amount of brown pigment in the eye. Carteolol is an antiglaucoma drug that may cause blurred vision, photophobia, and bradycardia. Dipivefrin is an antiglaucoma drug that may cause ocular discomfort and redness in the eye. Carbachol is an antiglaucoma drug that may cause transient ocular discomfort, headache, and blurred vision. p. 381

A patient reports, "While I was walking, I got something in my eye." What nursing intervention is most appropriate for a patient with a suspected foreign object in the eye? A. Beginning irrigation with sterile normal saline solution B. Attempting to remove the object without causing further damage to the eye C. Refraining from doing anything until the patient can be seen by an ophthalmologist D. Loosely covering the eye with a sterile patch and referring the patient to emergency care

D. Loosely covering the eye with a sterile patch and referring the patient to emergency care Rationale Covering the eye loosely with a sterile patch with referral for emergency care is the safest option for this patient. Eye irrigation and attempting to remove the object are not appropriate in this health care setting. The nurse should never attempt to remove a foreign object from the eye because this could cause further damage. The patient should be seen by the eye specialist, but covering the eye with an eye patch will prevent further trauma and irritation. p. 371

A patient informs the nurse that he or she is using a homemade saline solution to store contact lenses but is now having irritation and soreness. The nurse assesses inflammation of the cornea. On what medication does the nurse anticipate educating the patient? A. Tropicamide B. Besifloxacin C. Acetazolamide D. Polyhexamethylene biguanide

D. Polyhexamethylene biguanide Rationale Inflammation of the cornea indicates keratitis. Acanthamoeba keratitis is caused by a parasite that is associated with contact lens wear. Patients who use homemade saline solution are more susceptible to Acanthamoeba contamination. This organism is resistant to most drugs, and polyhexamethylene biguanide is an approved antifungal eye drop. Tropicamide is given during the preoperative phase to produce pupillary dilation. Besifloxacin is beneficial to a patient who has acute bacterial conjunctivitis. Acetazolamide decreases the aqueous humor production and is beneficial to a patient with glaucoma. p. 372

Which nursing intervention is the highest priority for a patient who has undergone retinal surgery? A. Monitoring the blood pressure B. Preventing fluid volume excess C. Maintaining a darkened environment D. Positioning and activity as preferred by surgeon

D. Positioning and activity as preferred by surgeon Rationale Postoperatively, the patient may be on bed rest and may require special positioning to maintain proper positioning to maintain proper position of an intravitreal bubble. The level of activity restriction after retinal surgery varies greatly, depending on the patient and surgeon. Monitoring blood pressure and preventing fluid volume excess are not necessarily related to post-retinal surgery care. Maintaining a darkened environment is not necessary and may present a risk for falling. p. 376

Prevention of vision loss resulting from chronic open-angle glaucoma is accomplished best by which intervention? A. Tobacco smoking cessation B. Yearly ophthalmic examination C. Eating a diet high in green leafy vegetables and lysine D. Strict adherence to prescribed eye drop medication schedule

D. Strict adherence to prescribed eye drop medication schedule Rationale Strict adherence to prescribed medication regimen to treat glaucoma will keep the intraoptic pressure at safe levels to avoid optic nerve damage. Tobacco cessation is healthy but will not treat glaucoma. Yearly eye examinations are important but will measure only any damage done if the patient does not follow treatment. A diet high in lutein, found in green leafy vegetables, is thought to improve eye health. Lysine is an amino acid that has some antiviral properties. pp. 379-380

A patient complains of a red, swollen, circumscribed, and acutely tender area in the lid margin. After the medical checkup, it is diagnosed as a hordeolum. The nurse should provide what instructions to help the patient care for the affected eye? A. Advise the patient to not perform lid scrubs for 10 days. B. Suggest wearing glasses to reduce development of the infection. C. Advise to avoid any treatment because the condition is a normal body protective mechanism. D. Suggest applying warm, moist compresses at least four times a day until the condition improves.

D. Suggest applying warm, moist compresses at least four times a day until the condition improves. Rationale Hordeolum is an infection caused by the bacteria Staphylococcus aureus. It manifests as a red, swollen, circumscribed, and acutely tender area in the lid margin. Treatment involves applying warm, moist compresses at least four times a day until the condition improves. If it recurs, lid scrubs should be performed daily to aid healing. In addition, appropriate antibiotic ointments or drops are prescribed if required. Wearing glasses may not help in treating hordeolum. It is not a normal body protective mechanism; therefore, treatment should not be deferred. p. 370

A patient is prescribed bimatoprost. What nursing intervention would ensure safe administration of the drug? A. Teach the patient to instill three drops three times a day. B. Advise the patient to wash hands with water and then instill the medicine. C. Suggest the patient rinse eyes immediately after instilling the medicine. D. Suggest the patient remove contact lenses 15 minutes before instilling the drops.

D. Suggest the patient remove contact lenses 15 minutes before instilling the drops. Rationale Bimatoprost is used in the treatment of glaucoma. The patient should be told to remove contact lenses 15 minutes before instilling the eyedrops. Instilling eyedrops with contact lenses on can cause a sensation of a foreign body in the eye. The patient should be instructed to instill one drop per evening in the eye. This is because increasing the dose results in increased brown iris pigmentation, ocular discomfort and redness, dryness, and itching. Instilling the medicine using aseptic techniques prevents infection. Washing eyes immediately after instilling the medicine may affect the therapeutic benefits. p. 381

A patient with glaucoma is taking timolol drops. What should the nurse include while reinforcing principles of medication administration with the patient? A. The patient will notice an improvement in vision within one month. B. The patient should use these on an as needed basis for eye irritation. C. The patient should maintain a supine position for 30 minutes after the drops are instilled. D. The patient may experience blurred vision after administration of the drops lasting several minutes.

D. The patient may experience blurred vision after administration of the drops lasting several minutes. Rationale It is common for patients to have a temporary blurring of vision for a few minutes after instilling eye drops. This should be no cause for concern to the patient. This medication should be used on a fixed schedule to reduce intraocular pressure. Glaucoma is asymptomatic and may not result in vision loss. It is not necessary to lie flat following eye drop administration. p. 381

The patient needs, but does not want, a corneal transplant because of difficulty with vision that may last for up to 12 months after the transplant. What can the nurse teach the patient about this? A. If the transplant is done soon after the donor dies, there will not be as much trouble recovering vision. B. The astigmatism the patient is experiencing may be corrected with glasses or rigid contact lenses. C. Increasing the amount of light and using a magnifier to read will be helpful if a transplant is not wanted. D. There are newer procedures in which only the damaged cornea's epithelial layer is replaced, and these procedures have a faster recovery.

D. There are newer procedures in which only the damaged cornea's epithelial layer is replaced, and these procedures have a faster recovery. Rationale The new procedures are called Descemet's stripping endothelial keratoplasty (DSEK) and Descemet's membrane endothelial keratoplasty (DMEK). Corneal transplants should be done as soon as possible, but this does not affect the rate of visual recovery. Astigmatism is not experienced with corneal scars and opacities requiring a corneal transplant. Increasing light and magnification helps a person with cataracts to read. p. 373

While preparing a patient with a visual problem for surgical therapy, the primary health care provider orders the nurse to administer cycloplegics to the patient. What rationale does the nurse identify for this action? A. To prevent anxiety B. To reduce inflammation C. To minimize photophobia D. To block the effect of acetylcholine on ciliary body muscles

D. To block the effect of acetylcholine on ciliary body muscles Rationale Cycloplegics are anticholinergic drugs that block the effect of acetylcholine on the ciliary body muscles and produce paralysis of accommodation. Therefore cycloplegics are given in the preoperative phase of surgical therapy to block the effect of acetylcholine on the iris sphincter muscle. Antianxiety drugs are given to the patient to prevent anxiety. Nonsteroidal antiinflammatory drugs are given to reduce inflammation. The patient is advised to wear dark glasses to minimize photophobia. p. 374


Kaugnay na mga set ng pag-aaral

Ch 24: The Digestive System Group 2: Sections 24.3 and 24.5 Dynamic Study Module

View Set

Decision Making and Problem Solving

View Set

Human Biology Quiz 1 Study Guide (Skeletal System)

View Set

Question 4-Explain 5 events that occur when a sarcomere contracts

View Set

Psych PrepU application style questions 51Q no ans choice w/exp

View Set